Saltar al contenido principal
LibreTexts Español

3.3: Pruebas de Convergencia

  • Page ID
    119228
  • \( \newcommand{\vecs}[1]{\overset { \scriptstyle \rightharpoonup} {\mathbf{#1}} } \) \( \newcommand{\vecd}[1]{\overset{-\!-\!\rightharpoonup}{\vphantom{a}\smash {#1}}} \)\(\newcommand{\id}{\mathrm{id}}\) \( \newcommand{\Span}{\mathrm{span}}\) \( \newcommand{\kernel}{\mathrm{null}\,}\) \( \newcommand{\range}{\mathrm{range}\,}\) \( \newcommand{\RealPart}{\mathrm{Re}}\) \( \newcommand{\ImaginaryPart}{\mathrm{Im}}\) \( \newcommand{\Argument}{\mathrm{Arg}}\) \( \newcommand{\norm}[1]{\| #1 \|}\) \( \newcommand{\inner}[2]{\langle #1, #2 \rangle}\) \( \newcommand{\Span}{\mathrm{span}}\) \(\newcommand{\id}{\mathrm{id}}\) \( \newcommand{\Span}{\mathrm{span}}\) \( \newcommand{\kernel}{\mathrm{null}\,}\) \( \newcommand{\range}{\mathrm{range}\,}\) \( \newcommand{\RealPart}{\mathrm{Re}}\) \( \newcommand{\ImaginaryPart}{\mathrm{Im}}\) \( \newcommand{\Argument}{\mathrm{Arg}}\) \( \newcommand{\norm}[1]{\| #1 \|}\) \( \newcommand{\inner}[2]{\langle #1, #2 \rangle}\) \( \newcommand{\Span}{\mathrm{span}}\)\(\newcommand{\AA}{\unicode[.8,0]{x212B}}\)

    Es muy común encontrar series para las que es difícil, o incluso prácticamente imposible, determinar exactamente la suma. A menudo se intenta evaluar la suma aproximadamente truncándolo, es decir, hacer que el índice se ejecute solo hasta algunos finitos\(N\text{,}\) en lugar de infinito. Pero no tiene sentido hacerlo si la serie diverge 1 2. Entonces te gusta al menos saber si la serie converge o diverge. Además también te gustaría saber qué error se introduce cuando aproximas\(\sum_{n=1}^\infty a_n\) por la “serie truncada”\(\sum_{n=1}^Na_n\text{.}\) Eso se llama el error de truncamiento. Hay una serie de “pruebas de convergencia” para ayudarte con esto.

    La prueba de divergencia

    Nuestra primera prueba es muy fácil de aplicar, pero también rara vez es útil. Simplemente nos permite rechazar rápidamente algunas series “trivialmente divergentes”. Se basa en la observación de que

    • por definición, una serie\(\sum_{n=1}^\infty a_n\) converge a\(S\) cuando las sumas parciales\(S_N=\sum_{n=1}^N a_n\) convergen a\(S\text{.}\)
    • Entonces, como\(N\rightarrow\infty\text{,}\) lo hemos hecho\(S_N\rightarrow S\) y, porque\(N-1\rightarrow\infty\) también, también tenemos\(S_{N-1}\rightarrow S\text{.}\)
    • Entonces\(a_N=S_N-S_{N-1}\rightarrow S-S=0\text{.}\)

    Esto nos dice que, si ya sabemos que una serie dada\(\sum a_n\) es convergente, entonces el\(n^{\rm th}\) término de la serie,\(a_n\text{,}\) debe converger\(0\) como\(n\) tiende al infinito. En esta forma, la prueba no es tan útil. Sin embargo, el contrapositivo 3 del enunciado es una prueba útil para la divergencia.

    Teorema 3.3.1 Prueba de Divergencia

    Si la secuencia\(\big\{a_n\big\}_{n=1}^\infty\) no logra converger a cero ya que\(n\rightarrow\infty\text{,}\) entonces la serie\(\sum_{n=1}^\infty a_n\) diverge.

    Ejemplo 3.3.2 Una simple divergencia

    Vamos\(a_n=\frac{n}{n+1}\text{.}\) Entonces

    \[ \lim_{n\rightarrow\infty} a_n =\lim_{n\rightarrow\infty}\frac{n}{n+1} =\lim_{n\rightarrow\infty}\frac{1}{1+\frac{1}{n}} =1\ne 0 \nonumber \]

    Entonces la serie\(\sum_{n=1}^\infty \frac{n}{n+1}\) diverge.

    Advertencia 3.3.3

    La prueba de divergencia es una “prueba unidireccional”. Nos dice que si\(\lim_{n\rightarrow\infty}a_n\) es distinto de cero, o no existe, entonces la serie\(\sum_{n=1}^\infty a_n\) diverge. Pero no nos dice absolutamente nada cuando\(\lim_{n\rightarrow\infty}a_n=0\text{.}\) En particular, es perfectamente posible que una serie diverja aunque\(\lim_{n\rightarrow\infty}a_n=0\text{.}\) Un ejemplo es\(\sum_{n=1}^\infty \frac{1}{n}\text{.}\) Vamos\(\sum_{n=1}^\infty a_n\) a mostrar en el Ejemplo 3.3.6, a continuación, que diverge.

    Ahora bien, si bien la convergencia o divergencia de series como se\(\sum_{n=1}^\infty \frac{1}{n}\) puede determinar usando algunos trucos inteligentes —véase el opcional §3.3.9—, sería mucho mejor tener métodos que sean más sistemáticos y se basen menos en ser astutos. En las siguientes subsecciones discutiremos varios métodos para probar series para la convergencia.

    Tenga en cuenta que si bien estas pruebas nos dirán si una serie converge o no, no nos dicen (salvo en casos raros) a qué suma la serie. Por ejemplo, la prueba que veremos en la siguiente subsección nos dice de inmediato que la serie

    \ begin {reunir*}\ sum_ {n=1} ^\ infty\ frac {1} {n^3}\ end {reunir*}

    converge. Sin embargo no nos dice su valor 4.

    La Prueba Integral

    En la prueba integral, pensamos en una serie\(\sum_{n=1}^\infty a_n\text{,}\) que no podemos evaluar explícitamente, como el área de una unión de rectángulos, con\(a_n\) representar el área de un rectángulo de ancho uno y alto\(a_n\text{.}\) Luego comparamos esa área con el área representada por una integral, que podemos evaluar explícitamente, tanto como lo hicimos en el Teorema 1.12.17, la prueba de comparación para integrales impropias. Empezaremos con un ejemplo sencillo, para ilustrar la idea. Después pasaremos a una formulación de la prueba en general.

    Ejemplo 3.3.4 Convergencia de la serie armónica

    Visualizar los términos de la serie armónica\(\sum_{n=1}^\infty\frac{1}{n}\) como un gráfico de barras — cada término es un rectángulo de altura\(\frac{1}{n}\) y anchura\(1\text{.}\) El límite de la serie es entonces el área limitante de esta unión de rectángulos. Considera el boceto de abajo a la izquierda.

    harmonic_int.svg

    Muestra que el área de las columnas sombreadas,\(\sum_{n=1}^4\frac{1}{n}\text{,}\) es mayor que el área bajo la curva\(y=\frac{1}{x}\) con\(1\le x\le 5\text{.}\) Eso es

    \ begin {alinear*}\ sum_ {n=1} ^4\ frac {1} {n} &\ ge\ int_1^5\ frac {1} {x}\, d {x}\ end {alinear*}

    Si tuviéramos que seguir dibujando las columnas hasta el infinito, entonces habríamos

    \ begin {alinear*}\ sum_ {n=1} ^\ infty\ frac {1} {n} &\ ge\ int_1^\ infty\ frac {1} {x}\, d {x}\ end {alinear*}

    Somos capaces de calcular esta integral inadecuada exactamente:

    \ begin {alinear*}\ int_1^\ infty\ frac {1} {x}\, d {x} &=\ lim_ {R\ a\ infty}\ grande [\ log|x|\ grande] _1^R = +\ infty\ end {align*}

    Es decir, el área bajo la curva diverge\(+\infty\) y así el área representada por las columnas también debe divergir para\(+\infty\text{.}\)

    Debe quedar claro que el argumento anterior puede generalizarse con bastante facilidad. Por ejemplo, el mismo argumento sostiene mutatis mutandis 5 para la serie

    \ begin {reunir*}\ sum_ {n=1} ^\ infty\ frac {1} {n^2}\ end {reunir*}

    En efecto vemos por el boceto de la derecha arriba que

    \ begin {alinear*}\ sum_ {n=2} ^N\ frac {1} {n^2} &\ le\ int_1^n\ frac {1} {x^2}\, d {x}\ end {alinear*}

    y por lo tanto

    \ comenzar {reunir*}\ sum_ {n=2} ^\ infty\ frac {1} {n^2}\ leq\ int_1^\ infty\ frac {1} {x^2}\, d {x}\ end {reunir*}

    Esta última integral impropia es fácil de evaluar:

    \ begin {alinear*}\ int_2^\ infty\ frac {1} {x^2}\, d {x} &=\ lim_ {R\ a\ infty}\ izquierda [-\ frac {1} {x}\ derecha] _2^R\\ &=\ lim_ {R\ a\ infty}\ izquierda (\ frac {1} {2} -\ frac ac {1} {R}\ derecha) =\ frac {1} {2}\ final {alinear*}

    Así sabemos que

    \ begin {reunir*}\ sum_ {n=1} ^\ infty\ frac {1} {n^2} = 1+\ sum_ {n=2} ^\ infty\ frac {1} {n^2}\ leq\ frac {3} {2}. \ end {reunir*}

    y así la serie debe converger.

    Los argumentos anteriores se formalizan en el siguiente teorema.

    Teorema 3.3.5 La Prueba Integral

    Dejar\(N_0\) ser cualquier número natural. Si\(f(x)\) es una función definida y continua para todos\(x\ge N_0\) y que obedece

    1. \(f(x)\ge 0\)para todos\(x\ge N_0\) y
    2. \(f(x)\)disminuye a medida que\(x\) aumenta y
    3. \(f(n)=a_n\)para todos\(n\ge N_0\text{.}\)

    intTest.svg

    Entonces

    \[ \sum_{n=1}^\infty a_n\text{ converges }\iff \int_{N_0}^\infty f(x)\ dx\text{ converges} \nonumber \]

    Además, cuando la serie converge, el error de truncamiento

    \[ \bigg|\sum_{n=1}^\infty a_n-\sum_{n=1}^N a_n\bigg|\le \int_N^\infty f(x)\ dx\qquad\text{for all $N\ge N_0$} \nonumber \]

    Prueba

    Dejar\(I\) ser cualquier entero fijo con\(I \gt N_0\text{.}\) Entonces

    • \(\sum_{n=1}^\infty a_n\)converge si y solo si\(\sum_{n=I}^\infty a_n\) converge — eliminar un número finito fijo de términos de una serie no puede afectar si converge o no.
    • Ya que\(a_n\ge 0\) para toda\(n\ge I \gt N_0\text{,}\) la secuencia de sumas parciales\(s_\ell=\sum_{n=I}^\ell a_n\) obedece Es\(s_{\ell+1} = s_\ell+a_{n+1} \ge s_\ell\text{.}\) decir,\(s_\ell\) aumenta a medida que\(\ell\) aumenta.
    • Así que\(\big\{s_\ell\big\}\) debe o bien converger a algún número finito o aumentar hasta el infinito. Es decir, o bien\(\sum_{n=I}^\infty a_n\) converge a un número finito o es\(+\infty\text{.}\)

    intTest3.svg

    Mira la figura anterior. El área sombreada en la figura es\(\sum_{n=I}^\infty a_n\)

    • porque el primer rectángulo sombreado tiene altura\(a_I\) y ancho\(1\text{,}\) y, por lo tanto, área\(a_I\) y
    • el segundo rectángulo sombreado tiene altura\(a_{I+1}\) y ancho\(1\text{,}\) y, por lo tanto, área\(a_{I+1}\text{,}\) y así sucesivamente

    Esta área sombreada es más pequeña que el área bajo la curva\(y=f(x)\) para\(I-1\le x \lt \infty\text{.}\) So

    \[ \sum_{n=I}^\infty a_n \le \int_{I-1}^\infty f(x)\ dx \nonumber \]

    y, si la integral es finita, la suma también\(\sum_{n=I}^\infty a_n\) es finita. Además, el límite deseado en el error de truncamiento es solo el caso especial de esta desigualdad con\(I=N+1\text{:}\)

    \ comenzar {reunir*}\ suma_ {n=1} ^\ infty a_n -\ suma_ {n=1} ^N a_n =\ suma_ {n=n+1} ^\ infty a_n\ le\ int_n^\ infty f (x)\ dx\ final {reunir*}

    intTest4.svg

    Para el “caso de divergencia” mira la figura anterior. El (nuevo) área sombreada en la figura es de nuevo\(\sum_{n=I}^\infty a_n\) porque

    • el primer rectángulo sombreado tiene altura\(a_I\) y ancho\(1\text{,}\) y, por lo tanto, área\(a_I\) y
    • el segundo rectángulo sombreado tiene altura\(a_{I+1}\) y ancho\(1\text{,}\) y, por lo tanto, área\(a_{I+1}\text{,}\) y así sucesivamente

    Esta vez el área sombreada es mayor que el área bajo la curva\(y=f(x)\) para\(I\le x \lt \infty\text{.}\) So

    \[ \sum_{n=I}^\infty a_n \ge \int_I^\infty f(x)\ dx \nonumber \]

    y, si la integral es infinita, la suma\(\sum_{n=I}^\infty a_n\) es infinita también.

    Ahora que tenemos la prueba integral, es sencillo determinar para qué valores de\(p\) la serie 6

    \ begin {reunir*}\ sum_ {n=1} ^\ infty\ frac {1} {n^p}\ end {reunir*}

    converge.

    Ejemplo 3.3.6 El\(p\) test: \(\sum\limits_{n=1}^\infty\frac{1}{n^p}\)

    Vamos Ahora\(p \gt 0\text{.}\) usaremos la prueba integral para determinar si la serie\(\sum_{n=1}^\infty\frac{1}{n^p}\) (que a veces se llama la\(p\) serie -series) converge o no.

    • Para ello, necesitamos una función\(f(x)\) que obedezca\(f(n)=a_n=\frac{1}{n^p}\) para todos\(n\) más grandes que algunos\(N_0\text{.}\) Ciertamente\(f(x)=\frac{1}{x^p}\) obedezca\(f(n)=\frac{1}{n^p}\) para todos\(n\ge 1\text{.}\) Así que escojamos esto\(f\) e intentemos\(N_0=1\text{.}\) (Siempre podemos aumentar\(N_0\) más tarde si es necesario.)
    • Esta función también obedece a las otras dos condiciones del Teorema 3.3.5:
      1. \(f(x) \gt 0\)para todos\(x\ge N_0=1\) y
      2. \(f(x)\)disminuye a medida que\(x\) aumenta porque\(f'(x)=-p\frac{1}{x^{p+1}} \lt 0\) para todos\(x\ge N_0=1\text{.}\)
    • Entonces la prueba integral nos dice que la serie\(\sum_{n=1}^\infty\frac{1}{n^p}\) converge si y sólo si la integral\(\int_1^\infty\frac{dx}{x^p}\) converge.
    • Ya hemos visto, en el Ejemplo 1.12.8, que la integral\(\int_1^\infty\frac{dx}{x^p}\) converge si y solo si\(p \gt 1\text{.}\)

    Entonces concluimos que\(\sum_{n=1}^\infty\frac{1}{n^p}\) converge si y solo si\(p \gt 1\text{.}\) Esto a veces se llama el\(p\) -test.

    • En particular, la serie\(\sum_{n=1}^\infty\frac{1}{n}\text{,}\) que se llama la serie armónica, tiene\(p=1\) y así diverge. A medida que sumamos más y más términos de esta serie juntos, los términos que agregamos, es decir\(\frac{1}{n}\text{,}\), se hacen cada vez más pequeños y tienden a cero, pero tienden a cero tan lentamente que la suma total sigue siendo infinita.
    • Por otro lado, la serie\(\sum_{n=1}^\infty\frac{1}{n^{1.000001}}\) tiene\(p = 1.000001 \gt 1\) y así converge. Esta vez a medida que sumamos más y más términos de esta serie juntos, los términos que agregamos, es decir,\(\frac{1}{n^{1.000001}}\text{,}\) tienden a cero (solo) lo suficientemente rápido como para que la suma total sea finita. Eso sí, para este ejemplo, la convergencia se da muy lentamente —hay que tomar una gran cantidad de términos para obtener una aproximación decente a la suma total. Si aproximamos\(\sum_{n=1}^\infty\frac{1}{n^{1.000001}}\) por la serie truncada\(\sum_{n=1}^N\frac{1}{n^{1.000001}}\text{,}\) cometemos un error de como máximo

      \ begin {alinear*}\ int_n^\ infty\ frac {dx} {x^ {1.000001}} & =\ lim_ {R\ rightarrow\ infty}\ int_n^R\ frac {dx} {x^ {1.000001}}\\ & =\ lim_ {R\ fila derecha\ infty} -\ frac {1} {0.000001} Grande [\ frac {1} {R^ {0.000001}} -\ frac {1} {N^ {0.000001}}\ Grande]\\ & =\ frac {10^6} {N^ {0.000001}}\ final {alinear*}

      Esto tiende a cero como\(N\rightarrow\infty\text{,}\) pero realmente lentamente.

    Ahora sabemos que la línea divisoria entre convergencia y divergencia de\(\sum_{n=1}^\infty\frac{1}{n^p}\) ocurre en\(p=1\text{.}\) Podemos profundizar un poco más y preguntarnos cuánto más rápido de lo que\(\frac{1}{n}\) el\(n^{\rm th}\) término necesita para encogerse para que la serie converja. Sabemos que para grandes\(x\text{,}\) la función\(\log x\) es más pequeña que\(x^a\) para cualquier positivo\(a\) —puedes convencerte de esto con una rápida aplicación de la regla de L'Hôpital. Por lo que no es irrazonable preguntar si la serie

    \ begin {reunir*}\ sum_ {n=2} ^\ infty\ frac {1} {n\ log n}\ end {reunir*}

    converge. Observe que sumamos de\(n=2\) porque cuando\(n=1, n\log n=0\text{.}\) Y no necesitamos detenernos ahí 7. Podemos analizar la convergencia de esta suma con cualquier potencia de\(\log n\text{.}\)

    Ejemplo 3.3.7\(\sum\limits_{n=2}^\infty\frac{1}{n(\log n)^p}\)

    Vamos Ahora\(p \gt 0\text{.}\) usaremos la prueba integral para determinar si la serie\(\sum\limits_{n=2}^\infty\frac{1}{n(\log n)^p}\) converge o no.

    • Como en el último ejemplo, comenzamos eligiendo una función que obedezca\(f(n)=a_n=\frac{1}{n(\log n)^p}\) para todos\(n\) más grandes que algunos\(N_0\text{.}\) Ciertamente\(f(x)=\frac{1}{x(\log x)^p}\) obedece\(f(n)=\frac{1}{n(\log n)^p}\) para todos\(n\ge 2\text{.}\) Así que usemos eso\(f\) e intentemos\(N_0=2\text{.}\)
    • Ahora vamos a revisar las otras dos condiciones del Teorema 3.3.5:
      1. Ambos\(x\) y\(\log x\) son positivos para todos\(x \gt 1\text{,}\) así que\(f(x) \gt 0\) para todos\(x\ge N_0=2\text{.}\)
      2. A\(x\) medida que aumenta tanto\(x\) y\(\log x\) aumenta y así\(x(\log x)^p\) aumenta y\(f(x)\) disminuye.
    • Entonces la prueba integral nos dice que la serie\(\sum\limits_{n=2}^\infty\frac{1}{n(\log n)^p}\) converge si y sólo si la integral\(\int_2^\infty\frac{dx}{x (\log x)^p}\) converge.
    • Para probar la convergencia de la integral, hacemos la sustitución\(u=\log x\text{,}\)\(du=\frac{dx}{x}\text{.}\)

      \ begin {reunir*}\ int_2^R\ frac {dx} {x (\ log x) ^p} =\ int_ {\ log 2} ^ {\ log R}\ frac {du} {u^p}\ end {reunión*}

      Ya sabemos que la integral la integral\(\int_1^\infty\frac{du}{u^p}\text{,}\) y por lo tanto la integral\(\int_2^R \frac{dx}{x (\log x)^p}\text{,}\) converge si y solo si\(p \gt 1\text{.}\)

    Entonces concluimos que\(\sum\limits_{n=2}^\infty\frac{1}{n(\log n)^p}\) converge si y solo si\(p \gt 1\text{.}\)

    La prueba de comparación

    Nuestra siguiente prueba de convergencia es la prueba de comparación. Se parece mucho a la prueba de comparación para integrales inadecuadas (ver Teorema 1.12.17) y es cierto por mucho las mismas razones. La idea aproximada es bastante simple. Una suma de términos mayores debe ser mayor que una suma de términos más pequeños. Entonces, si sabemos que la gran suma converge, entonces la pequeña suma debe converger también. Por otro lado, si sabemos que la pequeña suma diverge, entonces la suma grande también debe divergir. Formalizar esta idea da el siguiente teorema.

    Teorema 3.3.8 La prueba de comparación

    Dejar\(N_0\) ser un número natural y dejar\(K \gt 0\text{.}\)

    1. Si\(|a_n|\le K c_n\) por todos\(n\ge N_0\) y\(\sum\limits_{n=0}^\infty c_n\) converge, entonces\(\sum\limits_{n=0}^\infty a_n\) converge.
    2. Si\(a_n\ge K d_n\ge0 \) para todos\(n\ge N_0\) y\(\sum\limits_{n=0}^\infty d_n\) diverge, entonces\(\sum\limits_{n=0}^\infty a_n\) diverge.
    “Prueba”

    Aquí no vamos a probar este teorema. Simplemente observaremos que es muy razonable. Por eso hay comillas alrededor de “Prueba”. Para una prueba real ver la sección opcional 3.3.10.

    1. Si\(\sum\limits_{n=0}^\infty c_n\) converge a un número finito y si los términos en\(\sum\limits_{n=0}^\infty a_n\) son más pequeños que los términos en\(\sum\limits_{n=0}^\infty c_n\text{,}\) entonces no es de sorprender que\(\sum\limits_{n=0}^\infty a_n\) converja también.
    2. Si\(\sum\limits_{n=0}^\infty d_n\) diverge (es decir, suma a\(\infty\)) y si los términos en\(\sum\limits_{n=0}^\infty a_n\) son más grandes que los términos en\(\sum\limits_{n=0}^\infty d_n\text{,}\) entonces, por supuesto,\(\sum\limits_{n=0}^\infty a_n\) suma\(\infty\text{,}\) y así diverge, también.

    La prueba de comparación para series también se usa de la misma manera que la prueba de comparación para integrales inadecuadas. Por supuesto, se necesita una buena serie con la que comparar, y muchas veces la serie\(\sum n^{-p}\) (del Ejemplo 3.3.6), para algunos\(p \gt 0\text{,}\) resulta ser justo lo que se necesita.

    Ejemplo 3.3.9\(\sum_{n=1}^\infty\frac{1}{n^2+2n+3}\)

    Podríamos determinar si la serie\(\sum_{n=1}^\infty\frac{1}{n^2+2n+3}\) converge o no aplicando la prueba integral. Pero no vale la pena el esfuerzo 8. Si alguna serie converge o no está determinada por el comportamiento del summand 9 para muy grandes\(n\text{.}\) Así que el primer paso para abordar tal problema es desarrollar cierta intuición sobre el comportamiento de\(a_n\) cuando\(n\) es muy grande.

    • Paso 1: Desarrollar la intuición. En este caso, cuando\(n\) es muy grande 10\(n^2\gg 2n \gg 3\) para que ya\(\frac{1}{n^2+2n+3}\approx\frac{1}{n^2}\text{.}\) sepamos, del Ejemplo 3.3.6, eso\(\sum_{n=1}^\infty\frac{1}{n^p}\) converge si y solo si\(p \gt 1\text{.}\) Así\(\sum_{n=1}^\infty\frac{1}{n^2}\text{,}\) que que ha\(p=2\text{,}\) converge, y esperaríamos que eso\(\sum_{n=1}^\infty\frac{1}{n^2+2n+3}\) converja también.
    • Paso 2: Verificar la intuición. Podemos usar la prueba de comparación para confirmar que este es efectivamente el caso. Para cualquiera para que\(n\ge 1\text{,}\)\(n^2+2n+3 \gt n^2\text{,}\)\(\frac{1}{n^2+2n+3}\le\frac{1}{n^2}\text{.}\) Así que la prueba de comparación, Teorema 3.3.8, con\(a_n=\frac{1}{n^2+2n+3}\) y nos\(c_n=\frac{1}{n^2}\text{,}\) dice que\(\sum_{n=1}^\infty\frac{1}{n^2+2n+3}\) converge.

    Por supuesto que el ejemplo anterior fue “amañado” para dar una fácil aplicación de la prueba de comparación. A menudo es relativamente fácil, usando argumentos como los del Ejemplo 3.3.9, encontrar una serie “simple”\(\sum_{n=1}^\infty b_n\) con\(b_n\) casi lo mismo que\(a_n\) cuando\(n\) es grande. No obstante es bastante raro que\(a_n\le b_n\) para todos\(n\text{.}\) es mucho más común que\(a_n\le K b_n\) para alguna constante\(K\text{.}\) Esto es suficiente para permitir la aplicación de la prueba de comparación. Aquí hay un ejemplo.

    Ejemplo 3.3.10\(\sum_{n=1}^\infty\frac{n+\cos n}{n^3-1/3}\)

    Al igual que en el ejemplo anterior, el primer paso es desarrollar cierta intuición sobre el comportamiento de\(a_n\) cuando\(n\) es muy grande.

    • Paso 1: Desarrollar la intuición. Cuando\(n\) es muy grande,
      • \(n\gg |\cos n|\)para que el numerador\(n+\cos n\approx n\) y
      • \(n^3 \gg \frac{1}{3}\)para que el denominador\(n^3-\frac{1}{3}\approx n^3\text{.}\)

      Entonces cuando\(n\) es muy grande

      \[ a_n=\frac{n+\cos n}{n^3-\frac{1}{3}}\approx\frac{n}{n^3}=\frac{1}{n^2} \nonumber \]

      Ya sabemos por el Ejemplo 3.3.6, con\(p=2\text{,}\) eso\(\sum_{n=1}^\infty\frac{1}{n^2}\) converge, así que esperaríamos que eso\(\sum_{n=1}^\infty\frac{n+\cos n}{n^3-\frac{1}{3}}\) converja también.

    • Paso 2: Verificar la intuición. Podemos usar la prueba de comparación para confirmar que este es efectivamente el caso. Para ello necesitamos encontrar una constante\(K\) tal que sea menor que\(|a_n|= \frac{|n+\cos n|}{n^3-1/3}=\frac{n+\cos n}{n^3-1/3}\)\(\frac{K}{n^2}\) para todos\(n\text{.}\) Una buena manera 11 de hacerlo es factorizar el término dominante (en este caso\(n\)) fuera del numerador y también factorizar el término dominante (en este caso\(n^3\)) fuera del denominador.

      \[ a_n=\frac{n+\cos n}{n^3-\frac{1}{3}} =\frac{n}{n^3}\ \frac{1+\frac{\cos n}{n}}{1-\frac{1}{3n^3}} =\frac{1}{n^2}\ \frac{1+\frac{\cos n}{n}}{1-\frac{1}{3n^3}} \nonumber \]

      Entonces ahora necesitamos encontrar una constante\(K\) tal que\(\frac{1+\frac{(\cos n)}{n}}{1-\frac{1}{3n^3}}\) sea más pequeña que\(K\) para todos\(n\ge 1\text{.}\)

      • Primero considera el numerador\(1+(\cos n)\frac{1}{n}\text{.}\) Para todos\(n\ge 1\)
        • \(\frac{1}{n}\le 1\)y
        • \(|\cos n|\le 1\)

        Entonces el numerador siempre\(1+(\cos n)\frac{1}{n}\) es más pequeño que\(1+(1)\frac{1}{1}=2\text{.}\)

      • A continuación considera el denominador\(1-\frac{1}{3n^3}\text{.}\)
        • Cuando\(n\ge 1\text{,}\)\(\frac{1}{3n^3}\) se encuentra entre\(\frac{1}{3}\) y\(0\) para que
        • \(1-\frac{1}{3n^3}\)está entre\(\frac{2}{3}\) y\(1\) y en consecuencia
        • \(\frac{1}{1-\frac{1}{3n^3}}\)está entre\(\frac{3}{2}\) y\(1\text{.}\)
      • Como el numerador\(1+(\cos n)\frac{1}{n}\) es siempre más pequeño que\(2\) y siempre\(\frac{1}{1-\frac{1}{3n^3}}\) es menor que\(\frac{3}{2}\text{,}\) la fracción

        \[ \frac{1+\frac{\cos n}{n}}{1-\frac{1}{3n^3}} \le 2\Big(\frac{3}{2}\Big) =3 \nonumber \]

      Ahora sabemos que

      \[ |a_n| =\frac{1}{n^2}\ \frac{1+\frac{2}{n}}{1-\frac{1}{3n^3}} \le \frac{3}{n^2} \nonumber \]

      y, como sabemos\(\sum_{n=1}^\infty n^{-2}\) converge, la prueba de comparación nos dice que\(\sum_{n=1}^\infty\frac{n+\cos n}{n^3-1/3}\) converge.

    El último ejemplo fue en realidad una aplicación relativamente simple del teorema de comparación — encontrar una constante adecuada\(K\) puede ser realmente tedioso 12. Afortunadamente, existe una variante de la prueba de comparación que elimina por completo la necesidad de encontrar explícitamente\(K\text{.}\)

    La idea detrás de esto no es demasiado complicada. Ya hemos visto que la convergencia o divergencia de una serie depende no de sus primeros términos, sino solo de lo que sucede cuando\(n\) es realmente grande. En consecuencia, si podemos averiguar cómo se comportan los términos de la serie para realmente grandes\(n\) entonces podemos averiguar si la serie converge. Entonces, en lugar de comparar los términos de nuestra serie para todos\(n\text{,}\) solo compárelos cuando\(n\) sea grande.

    Teorema 3.3.11 Teorema de comparación de límites

    Dejar\(\sum_{n=1}^\infty a_n\) y\(\sum_{n=1}^\infty b_n\) ser dos series con\(b_n \gt 0\) para todos\(n\text{.}\) Supongamos que

    \[ \lim_{n\rightarrow\infty}\frac{a_n}{b_n}=L \nonumber \]

    existe.

    1. Si\(\sum_{n=1}^\infty b_n\) converge, entonces\(\sum_{n=1}^\infty a_n\) converge también.
    2. Si\(L\ne 0\) y\(\sum_{n=1}^\infty b_n\) diverge, entonces\(\sum_{n=1}^\infty a_n\) diverge también.

    En particular, si\(L\ne 0\text{,}\) entonces\(\sum_{n=1}^\infty a_n\) converge si y sólo si\(\sum_{n=1}^\infty b_n\) converge.

    Prueba

    a) Porque se nos dice que\(\lim_{n\rightarrow\infty}\frac{a_n}{b_n}=L \text{,}\) sabemos que,

    • cuando\(n\) es grande,\(\frac{a_n}{b_n}\) está muy cerca de\(L\text{,}\) por lo que\(\Big|\frac{a_n}{b_n}\Big|\) está muy cerca de\(|L|\text{.}\)
    • En particular, hay algún número natural\(\Big|\frac{a_n}{b_n}\Big|\le |L|+1\text{,}\) para\(N_0\) que para todos\(n\ge N_0\text{,}\) y por lo tanto
    • \(|a_n|\le Kb_n\)con\(K=|L|+1\text{,}\) para todos\(n\ge N_0\text{.}\)
    • El Teorema de comparación 3.3.8 implica ahora que\(\sum_{n=1}^\infty a_n\) converge.

    b) Supongamos que\(L \gt 0\text{.}\) (Si\(L \lt 0\text{,}\) sólo\(a_n\) reemplazamos por\(-a_n\text{.}\)) Porque se nos dice que\(\lim_{n\rightarrow\infty}\frac{a_n}{b_n}=L \text{,}\) sabemos que,

    • cuando\(n\) es grande,\(\frac{a_n}{b_n}\) está muy cerca de\(L\text{.}\)
    • En particular, hay algún número natural\(N\) para que\(\frac{a_n}{b_n}\ge \frac{L}{2}\text{,}\) y por lo tanto
    • \(a_n\ge Kb_n\)con\(K=\frac{L}{2} \gt 0\text{,}\) para todos\(n\ge N\text{.}\)
    • El Teorema de comparación 3.3.8 implica ahora que\(\sum_{n=1}^\infty a_n\) diverge.

    Los siguientes dos ejemplos ilustran cuánta mejora es el teorema anterior sobre la prueba de comparación recta (aunque, por supuesto, necesitábamos la prueba de comparación para desarrollar la prueba de comparación de límites).

    Ejemplo 3.3.12\(\sum_{n=1}^\infty\frac{\sqrt{n+1}}{n^2-2n+3}\)

    Set Primero\(a_n= \frac{\sqrt{n+1}}{n^2-2n+3}\text{.}\) tratamos de desarrollar alguna intuición sobre el comportamiento de\(a_n\) para grandes\(n\) y luego confirmamos que nuestra intuición era correcta.

    • Paso 1: Desarrollar la intuición. Cuando\(n\gg 1\text{,}\) el numerador\(\sqrt{n+1}\approx \sqrt{n}\text{,}\) y el denominador\(n^2-2n+3\approx n^2\) así\(a_n\approx \frac{\sqrt{n}}{n^2}=\frac{1}{n^{3/2}}\) y parece que nuestra serie debería converger por Ejemplo 3.3.6 con\(p=\frac{3}{2}\text{.}\)
    • Paso 2: Verificar la intuición. Para confirmar nuestra intuición establecemos\(b_n=\frac{1}{n^{3/2}}\) y calculamos el límite

      \[ \lim_{n\rightarrow\infty}\frac{a_n}{b_n} =\lim_{n\rightarrow\infty}\frac{\frac{\sqrt{n+1}}{n^2-2n+3}}{\frac{1}{n^{3/2}}} =\lim_{n\rightarrow\infty}\frac{n^{3/2}\sqrt{n+1}}{n^2-2n+3} \nonumber \]

      Nuevamente es una buena idea factorizar el término dominante fuera del numerador y el término dominante fuera del denominador.

      \[ \lim_{n\rightarrow\infty}\frac{a_n}{b_n} =\lim_{n\rightarrow\infty}\frac{n^2\sqrt{1+\frac{1}{n}}} {n^2\big(1-\frac{2}{n}+\frac{3}{n^2}\big)} =\lim_{n\rightarrow\infty}\frac{\sqrt{1+\frac{1}{n}}} {1-\frac{2}{n}+\frac{3}{n^2}} =1 \nonumber \]

      Ya sabemos que la serie\(\sum_{n=1}^\infty b_n =\sum_{n=1}^\infty\frac{1}{n^{3/2}}\) converge por Ejemplo 3.3.6 con\(p=\frac{3}{2}\text{.}\) Así nuestra serie converge por la prueba de comparación de límites, Teorema 3.3.11.
    Ejemplo 3.3.13\(\sum_{n=1}^\infty\frac{\sqrt{n+1}}{n^2-2n+3}\text{,}\) again

    También podemos tratar de tratar con la serie del Ejemplo 3.3.12, utilizando directamente la prueba de comparación. Pero eso nos obliga a encontrar\(K\) para que

    \ begin {alinear*}\ frac {\ sqrt {n+1}} {n^2-2n+3} &\ leq\ frac {K} {n^ {3/2}}\ end {align*}

    Podríamos hacer esto examinando el numerador y el denominador por separado:

    • El numerador no es tan malo ya que para todos\(n \geq 1\text{:}\)

      \ begin {align*} n+1 &\ leq 2n\ qquad\ text {y así}\\\ sqrt {n+1} &\ leq\ sqrt {2n}\ end {align*}

    • El denominador es un poco más complicado, ya que necesitamos un límite inferior, en lugar de un límite superior, y no podemos simplemente escribir\(|n^2-2n+3| \ge n^2\text{,}\) lo que es falso. En cambio tenemos que hacer un argumento más cuidadoso. En particular, nos gustaría encontrar\(N_0\) y\(K'\) para que\(n^2-2n+3\ge K'n^2\text{,}\) es decir,\(\frac{1}{n^2-2n+3}\le\frac{1}{K'n^2}\) para todos\(n \geq N_0\text{.}\) Para\(n\ge 4\text{,}\) nosotros tenemos\(2n = \frac{1}{2} 4n\le \frac{1}{2}n\cdot n=\frac{1}{2}n^2\text{.}\) Así que para\(n\ge 4\text{,}\)

      \ begin {alinear*} n^2-2n+3 &\ geq n^2 -\ frac {1} {2} n^2 + 3\ ge\ frac {1} {2} n^2\ end {alinear*}

    Al volver a juntar el numerador y el denominador tenemos

    \ begin {align*}\ frac {\ sqrt {n+1}} {n^2-2n+3} &\ leq\ frac {\ sqrt {2n}} {n^2/2} = 2\ sqrt {2}\ frac {1} {n^ {3/2}}\ qquad\ text {para todos $n\ ge 4$}\ end {align*}

    y la prueba de comparación nos dice entonces que nuestra serie converge. Es bastante claro que el enfoque del Ejemplo 3.3.12 fue mucho más sencillo.

    La prueba en serie alterna

    Cuando los signos de términos sucesivos en una serie se alternan entre\(+\) y\(-\text{,}\) como por ejemplo en\(\ 1-\frac{1}{2} +\frac{1}{3}-\frac{1}{4}+ \cdots\ \text{,}\) la serie se llama serie alterna. De manera más general, la serie

    \[ A_1-A_2+A_3-A_4+\cdots =\sum_{n=1}^\infty (-1)^{n-1} A_n \nonumber \]

    es alternando si cada\(A_n\ge 0\text{.}\) A menudo (pero no siempre) los términos en series alternas se vuelven sucesivamente más pequeños. Es decir, entonces\(A_1\ge A_2 \ge A_3 \ge \cdots\text{.}\) En este caso:

    • La primera suma parcial es\(S_1=A_1\text{.}\)
    • La segunda suma parcial,\(S_2=A_1-A_2\text{,}\) es menor que\(S_1\) por\(A_2\text{.}\)
    • La tercera suma parcial,\(S_3=S_2+A_3\text{,}\) es mayor que\(S_2\) por\(A_3\text{,}\) pero porque\(A_3\le A_2\text{,}\)\(S_3\) sigue siendo menor que\(S_1\text{.}\) Ver la figura a continuación.
    • La cuarta suma parcial,\(S_4=S_3-A_4\text{,}\) es menor que\(S_3\) por\(A_4\text{,}\) pero porque\(A_4\le A_3\text{,}\)\(S_4\) sigue siendo mayor que\(S_2\text{.}\) Otra vez, ver la figura a continuación.
    • Y así sucesivamente.

    Entonces las sucesivas sumas parciales oscilan, pero con amplitud siempre decreciente. Si, además,\(A_n\) tiende a\(0\) como\(n\) tiende a\(\infty\text{,}\) la amplitud de oscilación tiende a cero y la secuencia\(S_1\text{,}\)\(S_2\text{,}\)\(S_3\text{,}\)\(\cdots\) converge a algún límite\(S\text{.}\)

    Esto se ilustra en la figura

    altSeriesF.svg

    Aquí hay una prueba de convergencia para series alternas que explota esta estructura, y que es realmente fácil de aplicar.

    Teorema 3.3.14 Prueba de Serie Alternativa

    Dejar\(\big\{A_n\big\}_{n=1}^\infty\) ser una secuencia de números reales que obedece

    1. \(A_n\ge 0\)para todos\(n\ge 1\) y
    2. \(A_{n+1}\le A_n\)para todos\(n\ge 1\) (es decir, la secuencia es monótona decreciente) y
    3. \(\lim_{n\rightarrow\infty}A_n=0\text{.}\)

    Entonces

    \[ A_1-A_2+A_3-A_4+\cdots=\sum\limits_{n=1}^\infty (-1)^{n-1} A_n =S \nonumber \]

    converge y, por cada número natural\(N\text{,}\)\(S-S_N\) está entre\(0\) y (el primer término caído)\((-1)^N A_{N+1}\text{.}\) Aquí\(S_N\) está, como antes, la suma\(N^{\rm th}\) parcial\(\sum\limits_{n=1}^N (-1)^{n-1} A_n\text{.}\)

    “Prueba”

    Aquí sólo daremos parte de la prueba. Para el resto de la prueba ver la optativa sección 3.3.10. Arreglaremos cualquier número natural\(N\) y nos concentraremos en la última sentencia, lo que da un límite sobre el error de truncamiento (que es el error introducido cuando aproximas la serie completa por la suma parcial\(S_N\))

    \ begin {alinear*} E_N &= S-S_N=\ suma_ {n=n+1} ^\ infty (-1) ^ {n-1} a_N\ & = (-1) ^N\ Grande [A_ {N+1} -A_ {N+2} +A_ {N+3} -A_ {N+4} +\ cdots\ Grande]\ final {alinear*}

    Esta es por supuesto otra serie. Vamos a estudiar las sumas parciales

    \[ S_{N,\ell} = \sum_{n=N+1}^\ell (-1)^{n-1} A_n = (-1)^N\sum_{m=1}^{\ell-N} (-1)^{m-1} A_{N+m} \nonumber \]

    para esa serie.

    • Si\(\ell' \gt N+1\text{,}\) con\(\ell'-N\) par,\ comienza {alinear*} (-1) ^N S_ {N,\ ell'} &=\ overbrackets {(A_ {N+1} -A_ {N+2})} ^ {\ ge 0} +\ overbrackets {(A_ {N+3} -A_ {N+4})} ^ {\ ge 0} +\ cdots\\ &\ hskip1in+\ overbrackets {(A_ {\ ell'-1} -A_ {\ ell'})} ^ {\ ge 0}\\ &\ ge 0\\\\\ texto {y}\\\ (-1) ^N S_ {N,\ ell'+1} &=\ overbrackets {(-1) ^N S_ {N,\ ell'}} ^ {\ ge 0} +\ overbrackets {A_ {\ ell'+1}} ^ {\ ge 0}\ ge 0\ end {align*} Esto nos dice que\((-1)^N S_{N,\ell}\ge 0\) para todos\(\ell \gt N+1\text{,}\) tanto pares como impares.
    • Del mismo modo, si\(\ell' \gt N+1\text{,}\) con\(\ell'-N\) impar,

      \ begin {alinear*} (-1) ^N S_ {N,\ ell'} &=A_ {N+1} - (\ overbrackets {A_ {N+2} -A_ {N+3}} ^ {\ ge 0}) - (\ overbrackets {A_ {N+4} -A_ {N+5}} ^ {\ ge 0}) -\ cdots\\ &\ hskip1in -\ overbrackets {(A_ {\ ell'-1} -A_ {\ ell'})} ^ {\ ge 0}\\ &\ le A_ {N+1}\\ (-1) ^NS_ {N,\ ell'+1} &=\ overbrackets {(-1) ^N S_ {N,\ ell'}} ^ {\ le A_ {N+1}} -\ overcorsé {A_ {\ ell'+1}} ^ {\ ge 0}\ le A_ {N+1}\ final {alinear*}

      Esto nos dice que\((-1)^N S_{N,\ell}\le A_{N+1}\) para todos para todos\(\ell \gt N+1\text{,}\) tanto pares como impares.

    Entonces ahora sabemos que\(S_{N,\ell}\) se encuentra entre su primer mandato,\((-1)^NA_{N+1}\text{,}\) y\(0\) para todos\(\ell \gt N+1\text{.}\) Si bien no lo vamos a demostrar aquí (ver el apartado opcional 3.3.10), esto implica que, ya que\(A_{N+1}\rightarrow 0\) como\(N\rightarrow\infty\text{,}\) la serie converge y que

    \[ S-S_N=\lim_{\ell\rightarrow\infty} S_{N,\ell} \nonumber \]

    se encuentra entre\((-1)^NA_{N+1}\) y\(0\text{.}\)

    Ejemplo 3.3.15 Convergencia de la serie armónica alterna

    Ya hemos visto, en el Ejemplo 3.3.6, que la serie armónica\(\sum_{n=1}^\infty\frac{1}{n}\) diverge. Por otro lado, la serie\(\sum_{n=1}^\infty(-1)^{n-1}\frac{1}{n}\) converge por la prueba de series alternas con\(A_n=\frac{1}{n}\text{.}\) Note que

    1. \(A_n=\frac{1}{n}\ge 0\)para todos\(n\ge 1\text{,}\) por lo que\(\sum_{n=1}^\infty(-1)^{n-1}\frac{1}{n}\) realmente es una serie alterna, y
    2. \(A_n=\frac{1}{n}\)disminuye a medida que\(n\) aumenta, y
    3. \(\lim\limits_{n\rightarrow\infty}A_n =\lim\limits_{n\rightarrow\infty}\frac{1}{n}=0\text{.}\)

    de manera que se satisfagan todas las hipótesis de la prueba de series alternas, es decir, del Teorema 3.3.14. Veremos, en el Ejemplo 3.5.20, que

    \ begin {align*}\ sum_ {n=1} ^\ infty\ frac {(-1) ^ {n-1}} {n} &=\ log 2. \ end {alinear*}

    Ejemplo 3.3.16\(e\)

    Ya sabrás que\(e^x=\sum_{n=0}^\infty\frac{x^n}{n!} \text{.}\) En cualquier caso, lo demostraremos en el Ejemplo 3.6.5, a continuación. En particular

    \ comenzar {reunir*}\ frac {1} {e} =e^ {-1} =\ suma_ {n=0} ^\ infty\ frac {(-1) ^n} {n!} = 1 -\ frac {1} {1!} +\ frac {1} {2!} -\ frac {1} {3!} +\ frac {1} {4!} -\ frac {1} {5!} +\ cdots\ end {reunir*}

    es una serie alterna y satisface todas las condiciones de la prueba de series alternas, Teorema 3.3.14a:

    1. Los términos de la serie se alternan en signo.
    2. La magnitud del\(n^{\rm th}\) término en la serie disminuye monótonamente a medida que\(n\) aumenta.
    3. El\(n^{\rm th}\) término en la serie converge a cero como\(n\rightarrow\infty\text{.}\)

    Entonces la prueba en serie alterna garantiza que, si aproximamos, por ejemplo,

    \ begin {reunir*}\ frac {1} {e}\ approx\ frac {1} {2!} -\ frac {1} {3!} +\ frac {1} {4!} -\ frac {1} {5!} +\ frac {1} {6!} -\ frac {1} {7!} +\ frac {1} {8!} -\ frac {1} {9!} \ end {reunir*}

    entonces el error en esta aproximación se encuentra entre\(0\) y el siguiente término en la serie, que es\(\frac{1}{10!}\text{.}\) Eso es

    \ comenzar {reunir*}\ frac {1} {2!} -\ frac {1} {3!} +\ frac {1} {4!} -\ frac {1} {5!} +\ frac {1} {6!} -\ frac {1} {7!} +\ frac {1} {8!} -\ frac {1} {9!} \ le\ frac {1} {e}\ qquad\ qquad\ qquad\ qquad\\ qquad\\ qquad\ qquad\ qquad\ qquad\ le\ frac {1} {2!} -\ frac {1} {3!} +\ frac {1} {4!} -\ frac {1} {5!} +\ frac {1} {6!} -\ frac {1} {7!} +\ frac {1} {8!} -\ frac {1} {9!} +\ frac {1} {10!} \ end {reunir*}

    para que

    \ begin {reunir*}\ frac {1} {\ frac {1} {2!} -\ frac {1} {3!} +\ frac {1} {4!} -\ frac {1} {5!} +\ frac {1} {6!} -\ frac {1} {7!} +\ frac {1} {8!} -\ frac {1} {9!} +\ frac {1} {10!}} \ le e\ qquad\ qquad\ qquad\ qquad\\\ qquad\ qquad\ qquad\ qquad\ qquad\ le\ frac {1} {\ frac {1} {2!} -\ frac {1} {3!} +\ frac {1} {4!} -\ frac {1} {5!} +\ frac {1} {6!} -\ frac {1} {7!} +\ frac {1} {8!} -\ frac {1} {9!}} \ end {reunir*}

    que, a siete decimales dice

    \ begin {align*} 2.7182816\ le e\ le &2.7182837\ end {align*}

    (A siete decimales\(e=2.7182818\text{.}\))

    La prueba de series alternas nos dice que, para cualquier número natural\(N\text{,}\) el error que cometemos cuando nos aproximamos\(\frac{1}{e}\) por la suma parcial\(S_N= \sum_{n=0}^N\frac{(-1)^n}{n!}\) tiene una magnitud no mayor que\(\frac{1}{(N+1)!}\text{.}\) Esto tiende a cero espectacularmente rápidamente a medida que\(N\) aumenta, simplemente porque\((N+1)!\) aumenta espectacularmente rápido a medida que\(N\) aumenta 13. Por ejemplo\(20!\approx 2.4\times 10^{27}\text{.}\)

    Ejemplo 3.3.17 Computación\(\log\frac{11}{10}\)

    En breve veremos, en el Ejemplo 3.5.20, que si\(-1 \lt x\le 1\text{,}\) entonces

    \[ \log(1+x) = x-\frac{x^2}{2}+\frac{x^3}{3}-\frac{x^4}{4}+\cdots = \sum_{n=1}^\infty (-1)^{n-1}\frac{x^n}{n} \nonumber \]

    Supongamos que tenemos que\(\log\frac{11}{10}\) computar dentro de una precisión de\(10^{-12}\text{.}\) Ya que\(\frac{11}{10}=1+\frac{1}{10}\text{,}\) podemos obtener\(\log\frac{11}{10}\) evaluando\(\log(1+x)\) en\(x=\frac{1}{10}\text{,}\) para que

    \ begin {align*}\ log\ frac {11} {10} & =\ log\ Grande (1+\ frac {1} {10}\ Grande) =\ frac {1} {10} -\ frac {1} {2\ times 10^2} +\ frac {1} {3\ times 10^3} -\ frac {1} {4\ times 10^4} +\ cdots\\ & =\ sum_ {n=1} ^\ infty (-1) ^ {n-1}\ frac {1} {n\ times 10^n}\ end {align*}

    Por la prueba de series alternas, esta serie converge. También por la prueba de series alternas,\(\log\frac{11}{10}\) aproximándose desechando todos menos los primeros\(N\) términos

    \ begin {align*}\ log\ frac {11} {10} &\ approx\ frac {1} {10} -\ frac {1} {2\ times 10^2} +\ frac {1} {3\ times 10^3} -\ frac {1} {4\ times 10^4} +\ cdots + (-1) ^ {N-1}\ frac {1} N\ veces 10^N}\\ & =\ sum_ {n=1} ^ {N} (-1) ^ {n-1}\ frac {1} {n\ times 10^n}\ end {align*}

    introduce un error cuya magnitud no es más que la magnitud del primer término que tiramos.

    \[ \text{error} \le \frac{1}{(N+1)\times 10^{N+1}} \nonumber \]

    Para lograr un error que no sea más de lo que\(10^{-12}\text{,}\) tenemos que elegir para\(N\) que

    \[ \frac{1}{(N+1)\times 10^{N+1}} \le 10^{-12} \nonumber \]

    La mejor manera de hacerlo es simplemente adivinar —no vamos a ser capaces de manipular la desigualdad\(\frac{1}{(N+1)\times 10^{N+1}} \le \frac{1}{10^{12}}\) en la forma\(N\le \cdots\text{,}\) y aunque pudiéramos, no valdría la pena el esfuerzo. Tenemos que elegir para\(N\) que el denominador\((N+1)\times 10^{N+1}\) sea al menos\(10^{12}\text{.}\) Eso es fácil, porque el denominador contiene el factor\(10^{N+1}\) que es al menos\(10^{12}\) cuando es\(N+1\ge 12\text{,}\) decir, cuando sea\(N\ge 11\text{.}\) así lograremos un error de menos que\(10^{-12}\) si elegimos \(N=11\text{.}\)

    \[ \frac{1}{(N+1)\times 10^{N+1}}\bigg|_{N=11} = \frac{1}{12\times 10^{12}} \lt \frac{1}{10^{12}} \nonumber \]

    Esta no es la opción más pequeña posible de\(N\text{,}\) pero en la práctica eso simplemente no importa — a tu computadora no le va a importar si le pides o no que calcule algunos términos extra. Si realmente necesitas el más pequeño\(N\) que obedezca\(\frac{1}{(N+1)\times 10^{N+1}} \le \frac{1}{10^{12}}\text{,}\) puedes a continuación simplemente intentarlo\(N=10\text{,}\) entonces\(N=9\text{,}\) y así sucesivamente.

    \ begin {alinear*}\ frac {1} {(N+1)\ veces 10^ {N+1}}\ big|_ {N=11} &=\ frac {1} {12\ veces 10^ {12}}\ lt\ frac {1} {10^ {12}}\\ frac {1} {(N+1)\ veces 10^ {N+1}}\ big|_ {N=10} &=\ frac {1} {11\ veces 10^ {11}}\ lt\ frac {1} {10\ veces 10^ {11}} =\ frac {1} {10^ {12}}\\ frac {1} {(N+1)\ veces 10^ {N+1}}\ big|_ {N=9} &=\ frac {1} {10\ tiempos 10^ {10}} =\ frac {1} {10^ {11}}\ gt\ frac {1} {10^ {12}}\ end {align*}

    Entonces en este problema, el más pequeño aceptable\(N=10\text{.}\)

    La prueba de relación

    La idea detrás de la prueba de ratio proviene de un reexamen de la serie geométrica. Recordemos que la serie geométrica

    \ comenzar {reunir*}\ suma_ {n=0} ^\ infty a_n =\ suma_ {n=0} ^\ infty a r^n\ end {reunir*}

    converge cuando\(|r| \lt 1\) y diverge de otra manera. Entonces la convergencia de esta serie está completamente determinada por el número\(r\text{.}\) Este número es solo la relación de términos sucesivos —es decir,\(r = a_{n+1}/a_n\text{.}\)

    En general la relación de términos sucesivos de una serie, no\(\frac{a_{n+1}}{a_n}\text{,}\) es constante, sino que depende de\(n\text{.}\) Sin embargo, como hemos señalado anteriormente, la convergencia de una serie\(\sum a_n\) viene determinada por el comportamiento de sus términos cuando\(n\) es grande. De esta manera, el comportamiento de esta relación cuando\(n\) es pequeño no nos dice nada sobre la convergencia de la serie, sino del límite de la relación como\(n\to\infty\) lo hace. Esta es la base de la prueba de ratio.

    Prueba de Relación del Teorema 3.3.18

    Dejar\(N\) ser cualquier entero positivo y asumir que\(a_n\ne 0\) para todos\(n\ge N\text{.}\)

    1. Si\(\lim\limits_{n\rightarrow\infty}\Big|\frac{a_{n+1}}{a_n}\Big| = L \lt 1\text{,}\) entonces\(\sum\limits_{n=1}^\infty a_n\) converge.
    2. Si\(\lim\limits_{n\rightarrow\infty}\Big|\frac{a_{n+1}}{a_n}\Big| = L \gt 1\text{,}\) o\(\lim\limits_{n\rightarrow\infty}\Big|\frac{a_{n+1}}{a_n}\Big| = +\infty\text{,}\) luego\(\sum\limits_{n=1}^\infty a_n\) diverge.
    Advertencia 3.3.19

    Tenga en cuenta que la prueba de relación no proporciona absolutamente ninguna conclusión sobre la convergencia o divergencia de la serie\(\sum\limits_{n=1}^\infty a_n\) si\(\lim\limits_{n\rightarrow\infty}\Big|\frac{a_{n+1}}{a_n}\Big| = 1\text{.}\) Ver Ejemplo 3.3.22, a continuación.

    Prueba

    (a) Escoger cualquier número\(R\) obedeciendo\(L \lt R \lt 1\text{.}\) Estamos asumiendo que se\(\Big|\frac{a_{n+1}}{a_n}\Big|\) aproxima\(L\) como\(n\rightarrow\infty\text{.}\) En particular debe haber algún número\(M\) natural\(\Big|\frac{a_{n+1}}{a_n}\Big|\le R\) para que para todos\(n\ge M\text{.}\) Así que\(|a_{n+1}|\le R|a_n|\) para todos\(n\ge M\text{.}\) En particular

    \ begin {alinear*} |a_ {M+1} | &\\ le\ R\, |a_M|\\ |a_ {M+2} | &\\ le\ R\, |a_ {M+1} | &\ le\ R^2\, |a_m|\\ |a_ {M+3} | &\ le\ R\, |a_ {M+2} | & le\\ R^3\, |A_M|\\ &\ vdots\\ |a_ {M+\ ell} | &\ le R^\ ell\, |A_m|\ end {align*}

    para todos\(\ell\ge 0\text{.}\) La serie\(\sum_{\ell=0}^\infty R^\ell \,|a_M|\) es una serie geométrica con relación\(R\) menor a uno en magnitud y así converge. En consecuencia, por la prueba de comparación con\(a_n\) reemplazada por\(A_\ell = a_{n+\ell}\) y\(c_n\) reemplazada por\(C_\ell= R^\ell \, |a_M|\text{,}\) la serie\(\sum\limits_{\ell=1}^\infty a_{M+\ell} =\sum\limits_{n=M+1}^\infty a_n\) converge. Entonces la serie\(\sum\limits_{n=1}^\infty a_n\) converge también.

    (b) Estamos asumiendo que se\(\Big|\frac{a_{n+1}}{a_n}\Big|\) aproxima\(L \gt 1\) como\(n\rightarrow\infty\text{.}\) En particular debe haber algún número\(M \gt N\) natural\(\Big|\frac{a_{n+1}}{a_n}\Big|\ge 1\) para que para todos\(n\ge M\text{.}\) Así que\(|a_{n+1}|\ge |a_n|\) para todos Es\(n\ge M\text{.}\) decir,\(|a_n|\) aumente a medida que\(n\) aumente siempre y cuando\(n\ge M\text{.}\) Así\(|a_n|\ge |a_M|\) para todos \(n\ge M\)y\(a_n\) no puede converger a cero ya\(n\rightarrow\infty\text{.}\) que Así la serie diverge por la prueba de divergencia.

    Ejemplo 3.3.20\(\sum_{n=0}^\infty a n x^{n-1}\)

    Arreglar dos números reales distintos de cero\(a\) y ya\(x\text{.}\) hemos visto en el Ejemplo 3.2.4 y Lemma 3.2.5 —acabamos de\(r\) renombrar a\(x\) — que la serie geométrica\(\sum_{n=0}^\infty a x^n\) converge cuando\(|x| \lt 1\) y diverge cuando Ahora\(|x|\ge 1\text{.}\) vamos a considerar una nueva serie, construido diferenciando 14 cada término en la serie geométrica\(\sum_{n=0}^\infty a x^n\text{.}\) Esta nueva serie es

    \[ \sum_{n=0}^\infty a_n\qquad\text{with}\quad a_n = a\, n\, x^{n-1} \nonumber \]

    Apliquemos la prueba de ratio.

    \ begin {align*}\ Big|\ frac {a_ {n+1}} {a_n}\ Big| &=\ Big|\ frac {a\, (n+1)\, x^n} {a\, n\, x^ {n-1}}\ Big| =\ frac {n+1} {n} |x| =\ Grande (\ 1+ frac {1} {n}\ Grande) |x|\ fila derecha L=|x|\ quad\ texto {como $n\ fila derecha\ infty$}\ end {align*}

    La prueba de ratio ahora nos dice que la serie\(\sum_{n=0}^\infty a\, n\, x^{n-1}\) converge si\(|x| \lt 1\) y diverge si\(|x| \gt 1\text{.}\) No dice nada de los casos\(x=\pm 1\text{.}\) Pero en ambos de esos casos\(a_n=a\,n\,(\pm 1)^n\) no converge a cero como\(n\rightarrow\infty\) y la serie diverge por la prueba de divergencia.

    Observe que en el ejemplo anterior, tuvimos que aplicar otra prueba de convergencia además de la prueba de ratio. Esto será algo común cuando lleguemos a la serie power y a la serie Taylor — la prueba de ratio nos dirá algo así como

    La serie converge\(|x| \lt R\) y diverge para\(|x| \gt R\text{.}\)

    Por supuesto, todavía tendremos que determinar qué sucede cuando\(x=+R, -R\text{.}\) Para determinar convergencia o divergencia en esos casos necesitaremos usar una de las otras pruebas que hemos visto.

    Ejemplo 3.3.21\(\sum_{n=0}^\infty \frac{a}{n+1} X^{n + 1}\)

    Una vez más, arreglamos dos números reales distintos de cero\(a\) y nuevamente\(X\text{.}\) comenzamos con la serie geométrica\(\sum_{n=0}^\infty a x^n\) pero esta vez construimos una nueva serie integrando 15 cada término,\(a x^n\text{,}\) desde\(x=0\) hasta\(x=X\) dar\(\frac{a}{n+1} X^{n + 1}\text{.}\) La nueva serie resultante es

    \[ \sum_{n=0}^\infty a_n\qquad\text{with }a_n = \frac{a}{n+1} X^{n + 1} \nonumber \]

    Para aplicar la prueba de ratio necesitamos calcular

    \ begin {align*}\ Big|\ frac {a_ {n+1}} {a_n}\ Big| &=\ big|\ frac {\ frac {a} {n+2} X^ {n + 2}} {\ frac {a} {n+1} X^ {n + 1}}\ big| =\ frac {n+1} {n+2} |X| =\ frac {n+1} {n+2} |X| =\ frac {n+1} {n+2} |X| =\ frac ac {1+\ frac {1} {n}} {1+\ frac {2} {n}} |X|\ fila derecha L=|X|\ quad\ texto {como $n\ fila derecha\ infty$}\ end {alinear*}

    La prueba de ratio ahora nos dice que la serie\(\sum_{n=0}^\infty \frac{a}{n+1} X^{n + 1}\) converge si\(|X| \lt 1\) y diverge\(|X| \gt 1\text{.}\) si no dice nada de los casos\(X=\pm 1\text{.}\)

    Si\(X=1\text{,}\) la serie se reduce a

    \[ \sum_{n=0}^\infty \frac{a}{n+1} X^{n + 1}\bigg|_{X=1} =\sum_{n=0}^\infty \frac{a}{n+1} =a\sum_{m=1}^\infty \frac{1}{m}\qquad\text{with }m=n+1 \nonumber \]

    que es solo\(a\) veces la serie armónica, que sabemos diverge, por Ejemplo 3.3.6.

    Si\(X=-1\text{,}\) la serie se reduce a

    \[ \sum_{n=0}^\infty \frac{a}{n+1} X^{n + 1}\bigg|_{X=-1} =\sum_{n=0}^\infty (-1)^{n+1}\frac{a}{n+1} \nonumber \]

    que converge por la prueba en serie alterna. Ver Ejemplo 3.3.15.

    En conclusión, la serie\(\sum_{n=0}^\infty \frac{a}{n+1} X^{n + 1}\) converge si y solo si\(-1\le X \lt 1\text{.}\)

    La prueba de relación suele ser bastante fácil de aplicar, pero siempre se debe tener cuidado cuando el límite de la relación es\(1\text{.}\) El siguiente ejemplo ilustra esto.

    Ejemplo 3.3.22\(L=1\)

    En este ejemplo, vamos a ver tres series distintas que todas tienen\(\lim_{n\rightarrow\infty}\Big|\frac{a_{n+1}}{a_n}\Big| = 1\text{.}\) Uno va a divergir y las otras dos van a converger.

    • La primera serie es la serie armónica

      \[ \sum_{n=1}^\infty a_n\qquad\text{with }a_n = \frac{1}{n} \nonumber \]

      Ya hemos visto, en el Ejemplo 3.3.6, que esta serie diverge. Cuenta con

      \[ \Big|\frac{a_{n+1}}{a_n}\Big| = \bigg|\frac{\frac{1}{n+1}}{\frac{1}{n}}\bigg| = \frac{n}{n+1} = \frac{1}{1+\frac{1}{n}} \rightarrow L=1\quad\text{as $n\rightarrow\infty$} \nonumber \]

    • La segunda serie es la serie armónica alterna

      \[ \sum_{n=1}^\infty a_n\qquad\text{with }a_n = (-1)^{n-1}\frac{1}{n} \nonumber \]

      Ya hemos visto, en el Ejemplo 3.3.15, que esta serie converge. Pero también tiene

      \[ \Big|\frac{a_{n+1}}{a_n}\Big| = \bigg|\frac{(-1)^n\frac{1}{n+1}}{(-1)^{n-1}\frac{1}{n}}\bigg| = \frac{n}{n+1} = \frac{1}{1+\frac{1}{n}} \rightarrow L=1\quad\text{as $n\rightarrow\infty$} \nonumber \]

    • La tercera serie es

      \[ \sum_{n=1}^\infty a_n\qquad\text{with }a_n = \frac{1}{n^2} \nonumber \]

      Ya hemos visto, en el Ejemplo 3.3.6 con\(p=2\text{,}\) que esta serie converge. Pero también tiene

      \[ \Big|\frac{a_{n+1}}{a_n}\Big| = \bigg|\frac{\frac{1}{(n+1)^2}}{\frac{1}{n^2}}\bigg| = \frac{n^2}{(n+1)^2} = \frac{1}{(1+\frac{1}{n})^2} \rightarrow L=1\quad\text{as $n\rightarrow\infty$} \nonumber \]

    Hagamos un ejemplo algo artificial que nos obligue a combinar algunas de las técnicas que hemos visto.

    Ejemplo 3.3.23\(\sum_{n=1}^\infty \frac{ (-3)^n \sqrt{n+1}}{2n+3}x^n\)

    Nuevamente, la convergencia de esta serie dependerá de\(x\text{.}\)

    • Comencemos con la prueba de relación, así que calculamos\[\begin{align*} \left|\frac{a_{n+1}}{a_n}\right| &= \left|\frac{(-3)^{n+1} \sqrt{n+2} (2n+3) x^{n+1} }{(-3)^n \sqrt{n+1} (2n+5) x^n} \right|\\ &= |-3| \cdot \frac{\sqrt{n+2}}{\sqrt{n+1}} \cdot \frac{2n+3}{2n+5} \cdot |x|\\ \end{align*}\]

      Así que en el límite ya\(n \to \infty\) que nos quedamos con

      \ comenzar {alinear*}\ lim_ {n\ a\ infty}\ izquierda|\ frac {a_ {n+1}} {a_n}\ derecha| &= 3 |x|\ end {alinear*}
    • La prueba de relación nos dice entonces que si\(3|x| \gt 1\) la serie diverge, mientras que cuando\(3|x| \lt 1\) la serie converge.
    • Esto nos deja con los casos\(x=+\frac{1}{3}\) y\(-\frac{1}{3}\text{.}\)
    • Ajuste\(x=\frac{1}{3}\) da la serie

      \ comenzar {reunir*}\ sum_ {n=1} ^\ infty\ frac {(-1) ^n\ sqrt {n+1}} {2n+3}\ end {reunir*}

      El hecho de que los términos se alternen aquí sugiere que usamos la prueba de series alternas. Eso demostrará que esta serie converge siempre que\(\frac{\sqrt{n+1}}{2n+3}\) disminuya disminuye a medida que\(n\) aumenta. Así definimos la función

      \ begin {align*} f (t) &=\ frac {\ sqrt {t+1}} {2t+3}\ end {align*}

      (que se construye reemplazando el\(n\) in\(\frac{\sqrt{n+1}}{2n+3}\) con\(t\)) y verificar que\(f(t)\) es una función decreciente de\(t\text{.}\) Para probar eso, basta con mostrar que su derivada es negativa cuando\(t\geq 1\text{:}\)

      \ begin {align*} f' (t) &=\ frac {(2t+3)\ cdot\ frac {1} {2}\ cdot (t+1) ^ {-1/2} - 2\ sqrt {t+1}} {(2t+3) ^2}\\ &=\ frac {(2t+3) - 4 (t+1)} {2\ sqrt {t+1} (2t+3) ^2}\\ &=\ frac {-2t-1} {2\ sqrt {t+1} (2t+3) ^2}\ end {align*}

      Cuando\(t \geq 1\) esto es negativo y también lo\(f(t)\) es una función decreciente. Así podemos aplicar la prueba de series alternas para mostrar que la serie converge cuando\(x=\frac{1}{3}\text{.}\)
    • Cuando\(x = -\frac{1}{3}\) la serie se convierte en

      \ begin {reunir*}\ sum_ {n=1} ^\ infty\ frac {\ sqrt {n+1}} {2n+3}. \ end {reunir*}

      Observe que cuando\(n\) es grande, la suma es aproximadamente\(\frac{\sqrt{n}}{2n}\) lo que sugiere que la serie divergirá en comparación con\(\sum n^{-1/2}\text{.}\) Para formalizar esto, podemos usar el teorema de comparación de límites:

      \ begin {alinear*}\ lim_ {n\ a\ infty}\ frac {\ sqrt {n+1}} {2n+3}\\ frac {1} {n^ {-1/2}} &=\ lim_ {n\ a\ infty}\ frac {\ sqrt {n}\ cdot\ sqrt {1+1/n}} {n (2+3/n)}\ cdot\ sqrt {1+1/n}} {n (2+3/n)} punto n^ {1/2}\\ &=\ lim_ {n\ a\ infty}\ frac {n\ cdot\ sqrt {1+1/n}} {n (2+3/n)}\\ &=\ frac {1} {2}\ end {align*}

      Entonces como esta relación tiene un límite finito y la serie\(\sum n^{-1/2}\) diverge, sabemos que nuestra serie también diverge.

    Entonces, en resumen, la serie converge cuando\(-\frac{1}{3} \lt x \leq \frac{1}{3}\) y diverge de otra manera.

    Lista de pruebas de convergencia

    Ahora tenemos media docena de pruebas de convergencia:

    • Prueba de divergencia
      • funciona bien cuando el\(n^{\mathrm{th}}\) término en la serie no logra converger a cero ya que\(n\) tiende al infinito
    • Prueba en serie alterna
      • funciona bien cuando sucesivos términos de la serie se alternan en signo
      • no olvides comprobar que los términos sucesivos disminuyen en magnitud y tienden a cero como\(n\) tiende al infinito
    • Prueba Integral
      • funciona bien cuando, si\(x\) sustituyes\(n\) en el\(n^{\mathrm{th}}\) término obtienes una función,\(f(x)\text{,}\) que puedes integrar
      • no te olvides de verificar eso\(f(x)\ge 0\) y eso\(f(x)\) disminuye a medida que\(x\) aumenta
    • Prueba de relación
      • funciona bien cuando\(\frac{a_{n+1}}{a_n}\) simplifica lo suficiente como para que pueda calcular fácilmente\(\lim\limits_{n\rightarrow\infty}\big|\frac{a_{n+1}}{a_n}\big|=L\)
      • esto sucede a menudo cuando\(a_n\) contiene poderes, como\(7^n\text{,}\) o factoriales, como\(n!\)
      • no olvides que no\(L=1\) te dice nada sobre la convergencia/divergencia de la serie
    • Prueba de comparación y prueba de comparación de límites
      • funciona bien cuando, para muy grandes\(n\text{,}\) el\(n^{\mathrm{th}}\) término\(a_n\) es aproximadamente el mismo que un término más simple\(b_n\) (ver Ejemplo 3.3.10) y es fácil determinar si\(\sum_{n=1}^\infty b_n\) converge o no
      • no te olvides de comprobar que\(b_n\ge 0\)
      • generalmente la Prueba de Comparación de Límite es más fácil de aplicar que la Prueba de Comparación

    Opcional — La Torre Inclinada de los Libros

    Imagina que estás a punto de apilar un montón de libros idénticos sobre una mesa. Pero no quieres simplemente apilarlos exactamente verticalmente. Se quiere construir una “torre inclinada de libros” que sobresalga al borde de la mesa tanto como sea posible.

    bookStack.svg

    ¿Qué tan grande puede llegar a ser un voladizo? La respuesta a esa pregunta, que ahora derivaremos, ¡usa una serie!

    • Empecemos con sólo poner libro #1 sobre la mesa. Es el libro rojo etiquetado como “\(B_1\)” en la siguiente figura.
      bookStackV1.svg

      Utilice un\(x\) eje horizontal con\(x=0\) correspondiente al borde derecho de la mesa. Imagina que hemos colocado el libro #1 para que su borde derecho sobresalga el extremo de la mesa por una distancia\(x_1\text{.}\)

      • Para que el libro no se derrumbe de la mesa, necesitamos que su centro de masa quede sobre la mesa. Es decir, necesitamos la\(x\) coordenada de la masa central de la\(B_1\text{,}\) que denotaremos\(\bar X(B_1)\text{,}\) obedecer

        \[ \bar X(B_1) \le 0 \nonumber \]

        Asumiendo que nuestros libros tienen densidad uniforme y son de longitud\(L\text{,}\)\(\bar X(B_1)\) estará exactamente a medio camino entre el extremo derecho del libro, que está en\(x=x_1\text{,}\) y el extremo izquierdo del libro, que está en\(x=x_1-L\text{.}\) So

        \[ \bar X(B_1) =\frac{1}{2} x_1+\frac{1}{2}(x_1-L) = x_1-\frac{L}{2} \nonumber \]

      Así libro #1 no cae fuera de la mesa proporcionada

      \[ x_1\le\frac{L}{2} \nonumber \]

    • Ahora pongamos los libros #1 y #2 sobre la mesa, con el borde derecho del libro #1 en\(x=x_1\) y el borde derecho del libro #2 en\(x=x_2\text{,}\) como en la figura de abajo.
      bookStackV2a.svg
      • Para que el libro #2 no se derrumbe del libro #1, necesitamos que el centro de masa del libro #2 se encuentre sobre el libro #1. Es decir, necesitamos la\(x\) -coordenada de la masa central de la\(B_2\text{,}\) cual es\(\bar X(B_2)=x_2-\frac{L}{2}\text{,}\) obedecer

        \[ \bar X(B_2) \le x_1 \iff x_2-\frac{L}{2} \le x_1 \iff x_2\le x_1+\frac{L}{2} \nonumber \]

      • Suponiendo que el libro #2 no se derrupe del libro #1, todavía tenemos que arreglar que el par de libros no se vuelque de la mesa. Piense en el par de libros como el objeto rojo combinado en la figura
        bookStackV2b.svg

        Para que el objeto rojo combinado no se desprenda de la mesa, necesitamos que el centro de masa del objeto rojo combinado quede sobre la mesa. Es decir, necesitamos la\(x\) coordenada de la masa central del objeto rojo combinado, que denotaremos\(\bar X(B_1\cup B_2)\text{,}\) para obedecer

        \[ \bar X(B_1\cup B_2) \le 0 \nonumber \]

        El centro de masa del objeto rojo combinado es el promedio ponderado 16 de los centros de masa de\(B_1\) y\(B_2\text{.}\) As\(B_1\) y\(B_2\) tienen el mismo peso,

        \ begin {alinear*}\ bar X (B_1\ copa B_2) &=\ frac {1} {2}\ bar X (B_1) +\ frac {1} {2}\ bar X (B_2) =\ frac {1} {2}\ Grande (x_1-\ frac {L} {2}\ Grande) +\ frac {1} {2} Grande (x_2-\ frac {L} {2}\ Grande)\\ &=\ frac {1} {2} (x_1+x_2) -\ frac {L} {2}\ end {align*}

        y el objeto rojo combinado no se cae de la mesa si

        \[ \bar X(B_1\cup B_2) =\frac{1}{2}(x_1+x_2) -\frac{L}{2} \le 0 \iff x_1+x_2\le L \nonumber \]

      En conclusión, nuestra torre de dos libros sobrevive si

      \ begin {reunir*} x_2\ le x_1+\ frac {L} {2}\ qquad\ texto {y}\ qquad x_1+x_2\ le L\ end {reunir*}

      En particular podemos elegir\(x_1\) y\(x_2\) satisfacer\(x_2 = x_1+\frac{L}{2}\) y\(x_1+x_2 = L\text{.}\) Entonces, sustituyendo\(x_2 = x_1+\frac{L}{2}\) en\(x_1+x_2 = L\) da

      \[ x_1 + \Big(x_1+\frac{L}{2}\Big) = L \iff 2x_1 = \frac{L}{2} \iff x_1 = \frac{L}{2}\Big(\frac{1}{2}\Big),\quad x_2 = \frac{L}{2}\Big(1+\frac{1}{2}\Big) \nonumber \]

    • Antes de considerar la “torre\(n\) -libro” general, pongamos ahora los libros #1, #2 y #3 sobre la mesa, con el borde derecho del libro #1 en\(x=x_1\text{,}\) el borde derecho del libro #2 en\(x=x_2\text{,}\) y el borde derecho del libro #3 en\(x=x_3\text{,}\) como en la figura de abajo.
      bookStackV3a.svg
      • Para que el libro #3 no se derrumbe del libro #2, necesitamos que el centro de masa del libro #3 se encuentre sobre el libro #2. Es decir, necesitamos la\(x\) -coordenada de la masa central de la\(B_3\text{,}\) cual es\(\bar X(B_3)=x_3-\frac{L}{2}\text{,}\) obedecer

        \[ \bar X(B_3) \le x_2 \iff x_3-\frac{L}{2} \le x_2 \iff x_3\le x_2+\frac{L}{2} \nonumber \]

      • Asumiendo que el libro #3 no se derrama del libro #2, todavía tenemos que arreglar que el par de libros, libro #2 más libro #3 (el objeto rojo en la figura de abajo), no se vuelque del libro #1.
        bookStackV3b.svg

        Para que este objeto rojo combinado no se derrumbe del libro #1, necesitamos la\(x\) coordenada de su masa central, que denotamos\(\bar X(B_2\cup B_3)\text{,}\) para obedecer

        \[ \bar X(B_2\cup B_3) \le x_1 \nonumber \]

        El centro de masa del objeto rojo combinado es el promedio ponderado del centro de masas de\(B_2\) y\(B_3\text{.}\) As\(B_2\) y\(B_3\) tienen el mismo peso,

        \ begin {alinear*}\ bar X (B_2\ copa B_3) &=\ frac {1} {2}\ bar X (B_2) +\ frac {1} {2}\ bar X (B_3) =\ frac {1} {2}\ Grande (x_2-\ frac {L} {2}\ Grande) +\ frac {1} {2} Grande\ (x_3-\ frac {L} {2}\ Grande)\\ &=\ frac {1} {2} (x_2+x_3) -\ frac {L} {2}\ final {alinear*}

        y el objeto rojo combinado no se derriba del libro #1 si

        \[ \frac{1}{2}(x_2+x_3) -\frac{L}{2} \le x_1 \iff x_2+x_3\le 2x_1+L \nonumber \]

      • Asumiendo que el libro #3 no se derrama del libro #2, y también que el libro combinado #2 más libro #3 no se derrama del libro #1, todavía tenemos que arreglar que toda la torre de libros, libro #1 más libro #2 más libro #3 (el objeto rojo en la figura de abajo), no se derrama de la mesa.
        bookStackV3c.svg

        Para que este objeto rojo combinado no se desprenda de la mesa, necesitamos la\(x\) coordenada de su masa central, que denotamos\(\bar X(B_1\cup B_2\cup B_3)\text{,}\) para obedecer

        \[ \bar X(B_1\cup B_2\cup B_3) \le 0 \nonumber \]

        El centro de masa del objeto rojo combinado es el promedio ponderado del centro de masas de\(B_1\) y\(B_2\) y\(B_3\text{.}\) Como todos tienen el mismo peso,

        \ begin {alinear*}\ bar X (B_1\ copa B_2\ copa B_3) &=\ frac {1} {3}\ bar X (B_1) +\ frac {1} {3}\ bar X (B_2) +\ frac {1} {3}\ bar X (B_3)\ &=\ frac {1} {3}\ Grande (_1-\ frac {L} {2}\ Grande) +\ frac {1} {3}\ Grande (x_2-\ frac {L} {2}\ Grande) +\ frac {1} {3}\ Grande (x_3-\ frac {L} {2}\ Grande)\\ &=\ frac {1} {3} (x_1+x_2+x___3) -\ frac {L} {2}\ final {alinear*}

        y el objeto rojo combinado no se cae de la mesa si

        \[ \frac{1}{3}(x_1+ x_2+x_3) -\frac{L}{2} \le 0 \iff x_1+ x_2+x_3\le \frac{3L}{2} \nonumber \]

      En conclusión, nuestra torre de tres libros sobrevive si

      \ begin {reunir*} x_3\ le x_2+\ frac {L} {2}\ qquad\ texto {y}\ qquad x_2+x_3\ le 2x_1 + L\ qquad\ texto {y}\ qquad x_1+ x_2+x_3\ le\ frac {3L} {2}\ end {reunir*}

      En particular, podemos elegir\(x_1\text{,}\)\(x_2\) y\(x_3\) satisfacer

      \ begin {align*} x_1+ x_2+x_3&=\ frac {3L} {2}\ qquad\ texto {y}\\ x_2+x_3&= 2x_1 + L\ qquad\ texto {y}\\ x_3 &=\ frac {L} {2} + x_2\ end {align*}

      Sustituir la segunda ecuación por la primera da

      \ begin {reunir*} 3x_1 +L =\ frac {3L} {2}\ implica x_1 =\ frac {L} {2}\ Grande (\ frac {1} {3}\ Grande)\ end {reunir*}

      Luego sustituyendo la tercera ecuación por la segunda, y luego usando la fórmula anterior para\(x_1\text{,}\) da

      \ begin {reunir*} 2x_2 +\ frac {L} {2} = 2x_1+L =\ frac {L} {3} + L\ implica x_2 =\ frac {L} {2}\ Grande (\ frac {1} {2} +\ frac {1} {3}\ Grande)\ end {reunión*}

      y finalmente

      \ begin {reunir*} x_3 =\ frac {L} {2} + x_2 =\ frac {L} {2}\ Grande (1+\ frac {1} {2} +\ frac {1} {3}\ Grande)\ end {reunir*}

    • Por fin estamos listos para la “torre del\(n\) libro” general. Apilar\(n\) libros sobre la mesa, con libro\(B_1\) en la parte inferior y libro\(B_n\) en la parte superior, y con el borde derecho del libro #\(j\) en\(x=x_j\text{.}\) Las mismas consideraciones de centro de masa que las anteriores muestran que la torre sobrevive si

      \ begin {alinear*}\ bar X (b_n) &\ le x_ {n-1} & x_n-\ frac {L} {2} &\ le x_ {n-1}\\ bar X (B_ {n-1}\ copa b_n) &\ le x_ {n-2} &\ frac {1} {2} (x_ {n-1} +x_n) -\ frac {L} {2} &\ le x_ {n-2}\\ &\\ vdots &\ quad\ vdots\\ vdots\\ bar X (B_3\ taza\ cdots\ taza B_n) &\ le x_2&\ frac {1} {n-2} (x_3+\ cdots+x_n) -\ frac {L} {2} & ;\ le x_2\\\ bar X (B_2\ copa B_3\ taza\ cdots\ taza B_n) &\ le x_1&\ frac {1} {n-1} (x_2+x_3+\ cdots+x_n) -\ frac {L} {2} &\ le x_1\\ bar X (B_1\ taza B_2 taza\ _3\ copa\ cdots\ copa b_n) &\ le 0 &\ frac {1} {n} (x_1+x_2+x_3+\ cdots+x_n) -\ frac {L} {2} &\ le 0\ end {align*}

      En particular, podemos elegir los\(x_j\)'s para obedecer

      \ begin {alinear*}\ frac {1} {n} (x_1+x_2+x_3+\ cdots+x_n) & =\ frac {L} {2}\\ frac {1} {n-1} (x_2+x_3+\ cdots+x_n) &=\ frac {L} {2} + x_1\\ frac {1} {n-2} (x_3+\ cdots+x_n) &=\ frac {L} {2} + x_2\\ &\\ vdots &\ vdots&\\\ frac {1} {2} (x_ {n-1} +x_n) &=\ frac {L} {2} + x_ {n-2}\\ x_n=\ frac {L} {2} + x_ {n-1}\ fin {alinear*}

      Sustituir\(x_2+x_3+\cdots+x_n=(n-1) x_1 +\frac{L}{2}(n-1)\) de la segunda ecuación a la primera ecuación da

      \ begin {alinear*}\ frac {1} {n}\ Grande\ {\ overbrackets {x_1+ (n-1) x_1} ^ {nx_1} +\ frac {L} {2} (n-1)\ Grande\} =\ frac {L} {2} &\ implica x_1 +\ frac {L} {2}\ Grande (1-\ frac {1} {n}\ Grande) =\ frac {L} {2}\ Grande (\ frac {1} {2}\ Grande)\\ &\ implica x_1 =\ frac {L} {2}\ Grande (\ frac {1} {n}\ Grande)\ end {align*}

      Sustituir\(x_3+\cdots+x_n=(n-2) x_2+\frac{L}{2}(n-2)\) de la tercera ecuación a la segunda ecuación da

      \ begin {alinear*} &\ frac {1} {n-1}\ Grande\ {\ overbrackets {x_2+ (n-2) x_2} ^ {(n-1) x_2} +\ frac {L} {2} (\ overbrackets {n-2} ^ {(n-1) -1})\ Grande\} =\ frac {L} {2} +x_1 =\ frac {L} {2}\ Grande (1+\ frac {1} {n}\ Grande)\\ &\ hskip1in\ implica x_2 +\ frac {L} {2}\ Grande (1-\ frac {1} {n-1}\ Grande) =\ frac {L} {2}\ Grande (1+\ frac {1} {n}\ Grande)\ &\ hskskip1in\ implica x_2 =\ frac {L} {2}\ Grande (\ frac {1} {n-1} +\ frac {1} {n}\ Grande)\ end {align*}

      Sólo sigue adelante. Terminamos con

      \ begin {align*} x_1 &=\ frac {L} {2}\ Grande (\ frac {1} {n}\ Grande)\\ x_2 &=\ frac {L} {2}\ Grande (\ frac {1} {n-1} +\ frac {1} {n}\ Grande)\ x_3 &=\ frac {L} {2} Grande\ (\ frac {1} {n-2} +\ frac {1} {n-1} +\ frac {1} {n}\ Grande)\\ &\\ vdots\\ x_ {n-2} &=\ frac {L} {2}\ Grande (\ frac {1} {3} +\ cdots+\ frac {1} {n}\ Grande)\ x_ {n-1} &=\ frac {L} {2}\ Grande (\ frac {1} {2} +\ frac {1} {3} +\ cdots+\ frac {1} {n}\ Grande)\\ x_n &=\ frac {L} {2}\ Grande (1+\ frac {1} {2} +\ frac {1} {3} +\ cdots+\ frac {1} {n}\ Grande)\ end {align*}

      Nuestro voladizo es\(x_n = \frac{L}{2}\big(1+\frac{1}{2}+\frac{1}{3}+\cdots+\frac{1}{n}\big)\text{.}\) Esto es\(\frac{L}{2}\) veces la suma\(n^{\rm th}\) parcial de la serie armónica\(\sum_{m=1}^\infty\frac{1}{m}\text{.}\) Como vimos en el Ejemplo 3.3.6 (la\(p\) prueba), la serie armónica diverge. Entonces, como\(n\) va al infinito\(1+\frac{1}{2}+\frac{1}{3}+\cdots+\frac{1}{n}\) también va al infinito. ¡Podemos hacer el voladizo tan grande 17 como queramos!

    Opcional — La Prueba Raíz

    Hay otra prueba que es muy similar en espíritu a la prueba de relación. También proviene de un reexamen de la serie geométrica

    \ comenzar {reunir*}\ suma_ {n=0} ^\ infty a_n =\ suma_ {n=0} ^\ infty a r^n\ end {reunir*}

    La prueba de relación se basó en la observación de\(r\text{,}\) que lo que determina en gran medida si la serie converge o no, se pudo encontrar calculando\(r = a_{n+1}/a_n\text{.}\) la relación La prueba raíz se basa en la observación que también se\(|r|\) puede determinar al mirar que la\(n^{\rm th}\) raíz del\(n^{\rm th}\) término con\(n\) muy grande:

    \[ \lim_{n\to\infty}\root{n}\of{\big|ar^n\big|} =|r|\lim_{n\to\infty}\root{n}\of{\big|a\big|} =|r|\qquad\text{if $a\ne 0$} \nonumber \]

    Por supuesto, en general, el\(n^{\rm th}\) término no es exactamente\(ar^n\text{.}\) Sin embargo, si por muy grande\(n\text{,}\) el\(n^{\rm th}\) término es aproximadamente proporcional a\(r^n\text{,}\) con\(|r|\) dado por el límite anterior, esperaríamos que la serie converja cuando\(|r| \lt 1\) y diverja cuando\(|r| \gt 1\text{.}\) Eso es en efecto el caso.

    Teorema 3.3.24 Prueba Raíz

    Supongamos que

    \[ L = \lim_{n\to\infty}\root{n}\of{\big|a_n\big|} \nonumber \]

    existe o es\(+\infty\text{.}\)

    1. Si\(L \lt 1\text{,}\) entonces\(\sum\limits_{n=1}^\infty a_n\) converge.
    2. Si\(L \gt 1\text{,}\) o\(L=+\infty\text{,}\) luego\(\sum\limits_{n=1}^\infty a_n\) diverge.
    Advertencia 3.3.25

    Tenga en cuenta que la prueba raíz no proporciona absolutamente ninguna conclusión sobre la convergencia o divergencia de la serie\(\sum\limits_{n=1}^\infty a_n\) si\(\lim\limits_{n\rightarrow\infty}\root{n}\of{\big|a_n\big|} = 1\text{.}\)

    Prueba

    (a) Escoger cualquier número\(R\) obedeciendo\(L \lt R \lt 1\text{.}\) Estamos asumiendo que se\(\root{n}\of{|a_n|}\) acerca\(L\) como\(n\rightarrow\infty\text{.}\) En particular debe haber algún número\(M\) natural\(\root{n}\of{|a_n|}\le R\) para que para todos\(n\ge M\text{.}\) Así que\(|a_n|\le R^n\) para todos\(n\ge M\) y la serie\(\sum\limits_{n=1}^\infty a_n\) converja en comparación con la serie geométrica\(\sum\limits_{n=1}^\infty R^n\)

    (b) Estamos asumiendo que se\(\root{n}\of{|a_n|}\) acerca\(L \gt 1\) (o crece sin límites) como\(n\rightarrow\infty\text{.}\) En particular debe haber algún número\(M\) natural\(\root{n}\of{|a_n|}\ge 1\) para que para todos\(n\ge M\text{.}\) Así\(|a_n|\ge 1\) para todos\(n\ge M\) y la serie diverja por la prueba de divergencia.

    Ejemplo 3.3.26\(\sum_{n=1}^\infty \frac{ (-3)^n \sqrt{n+1}}{2n+3}x^n\)

    Ya hemos utilizado la prueba de ratio, en el Ejemplo 3.3.23, para mostrar que esta serie converge cuando\(|x| \lt \frac{1}{3}\) y diverge cuando Ahora\(|x| \gt \frac{1}{3}\text{.}\) usaremos la prueba raíz para sacar las mismas conclusiones.

    • Escribir\(a_n= \frac{ (-3)^n \sqrt{n+1}}{2n+3}x^n\text{.}\)
    • Nosotros computamos

      \ begin {align*}\ root {n}\ de {|a_n|} &=\ root {n}\ de {\ bigg|\ frac {(-3) ^n\ sqrt {n+1}} {2n+3} x^n\ bigg|}\\ &= 3 |x|\ grande (n+1\ grande) ^ {\ frac {1} {2n}}\ grande (2n+3) ^ {-\ frac {1} {n}}\ final {alinear*}

    • Ahora vamos a mostrar que el límite de\(\big(n+1\big)^{\frac{1}{2n}}\) como\(n\to\infty\) es exactamente\(1\text{.}\) Para hacer, así que primero calculamos el límite del logaritmo.

      \ begin {alinear*}\ lim_ {n\ a\ infty}\ log\ big (n+1\ big) ^ {\ frac {1} {2n}} &=\ lim_ {n\ a\ infty}\ frac {\ log\ big (n+1\ big)} {2n}\ qquad&\ text {ahora aplica Teorema} {\ text {3.1.6}\ &=\ lim_ {t\ a\ infty}\ frac {\ log\ grande (t+1\ grande)} {2t}\\ &=\ lim_ {t\ a\ infty}\ frac {\ frac {1} {t+1}} {2}\ qquad&\ text {por l'Hôpital}\\ &=0 \ end {alinear*}

      Entonces

      \ begin {reunir*}\ lim_ {n\ a\ infty}\ grande (n+1\ grande) ^ {\ frac {1} {2n}} =\ lim_ {n\ a\ infty}\ exp\ grande\ {\ log\ grande (n+1\ grande) ^ {\ frac {1} {2n}}\ grande\} = e^0=1\ end {recolecta} *

      Un cálculo esencialmente idéntico también da que\(\lim_{n\to\infty}\big(2n+3)^{-\frac{1}{n}} = e^0=1\text{.}\)
    • Entonces

      \ comenzar {reunir*}\ lim_ {n\ a\ infty}\ raíz {n}\ de {|a_n|} = 3 |x|\ end {reunir*}

    y la prueba raíz también nos dice que si\(3|x| \gt 1\) la serie diverge, mientras que cuando\(3|x| \lt 1\) la serie converge.

    Hemos hecho el último ejemplo una vez, en el Ejemplo 3.3.23, usando la prueba de ratio y una vez, en el Ejemplo 3.3.26, usando la prueba raíz. Claramente fue mucho más fácil usar la prueba de ratio. Aquí hay un ejemplo que es manejado más fácilmente por la prueba raíz.

    Ejemplo 3.3.27\(\sum_{n=1}^\infty \big(\frac{n}{n+1}\big)^{n^2}\)

    Escribe\(a_n= \big(\frac{n}{n+1}\big)^{n^2}\text{.}\) Entonces

    \ begin {align*}\ root {n}\ de {|a_n|} &=\ root {n}\ de {\ Grande (\ frac {n} {n+1}\ Grande) ^ {n^2}} =\ Grande (\ frac {n} {n+1}\ Grande) ^ {n} =\ Grande (1+\ frac {1} {n}\ Grande) ^ {-n}\ end {alinear*}

    Ahora tomamos el límite,

    \ begin {alinear*}\ lim_ {n\ a\ infty}\ Grande (1+\ frac {1} {n}\ Grande) ^ {-n} &=\ lim_ {X\ a\ infty}\ Grande (1+\ frac {1} {X}\ Grande) ^ {-X}\ qquad&\ text {por teorema} {\ text {3.1.6}}\ =\ lim_ {x\ a 0}\ grande (1+x\ grande) ^ {-1/x}\ qquad&\ text {donde $x=\ frac {1} {X} $}\\ &= e^ {-1}\ end {align*}

    por Ejemplo 3.7.20 en el texto CLP-1 con\(a=-1\text{.}\) As el límite es estrictamente más pequeño que\(1\text{,}\) la serie\(\sum_{n=1}^\infty \big(\frac{n}{n+1}\big)^{n^2}\) converge.

    Para sacar la misma conclusión usando la prueba de ratio, se tendría que demostrar que el límite de

    \ begin {align*}\ frac {a_ {n+1}} {a_n} &=\ Grande (\ frac {n+1} {n+2}\ Grande) ^ {(n+1) ^2}\ Grande (\ frac {n+1} {n}\ Grande) ^ {n^2}\ end {align*}

    ya\(n\rightarrow\infty\) que es estrictamente menor que 1. Claramente es mejor seguir con la prueba de raíz.

    Opcional — Serie Harmonic y Basel

    La serie armónica

    La serie

    \ comenzar {reunir*}\ sum_ {n=1} ^\ infty\ frac {1} {n}\ fin {reunir*}

    que apareció en Warning 3.3.3, se llama la serie Harmonic 18, y sus sumas parciales

    \ begin {align*} H_N &=\ sum_ {n=1} ^N\ frac {1} {n}\ end {align*}

    se llaman los números armónicos. Aunque estos números han sido estudiados al menos desde Pitágoras, la divergencia de la serie fue probada por primera vez alrededor de 1350 por Nicholas Oresme (1320-5 — 1382), aunque la prueba se perdió durante muchos años y fue redescubierta por Mengoli (1626—1686) y los hermanos Bernoulli (Johann 1667—1748 y Jacob 1655— 1705).

    La prueba de Oresme es hermosa y aún más notable que se produjo más de 300 años antes de que Newton y Leibnitz desarrollaran el cálculo. Comienza agrupando cuidadosamente los términos de la serie armónica:

    \ begin {alinear*} &\ sum_ {n=1} ^\ infty\ frac {1} {n} = 1 +\ frac {1} {2} +\ frac {1} {3} +\ frac {1} {4} +\ frac {1} {5} +\ frac {1} {6} +\ frac {1} {7} +\ frac {1} {8} +\ cdots\\ &= 1 +\ frac {1} {2} +\ izquierda (\ frac {1} {3} +\ frac {1} {4}\ derecha) +\ izquierda (\ frac {1} {5} +\ frac {1} {6} +\ frac {1} {7} +\ frac {1} {8} derecha) +\ izquierda (\ frac {1} {9} +\ frac {1} {10} +\ cdots +\ frac {1} {15} +\ frac {1} {16}\ derecha) +\ cdots\\ &\ gt 1 +\ frac {1} {2} +\ izquierda (\ frac {1} {4} +\ frac {1} {4}\ derecha) + izquierda\ (\ frac {1} {8} +\ frac {1} {8} +\ frac {1} {8} +\ frac {1} {8}\ derecha) +\ izquierda (\ frac {1} {16} +\ frac {1} {16} +\ cdots +\ frac {1} {16} +\ frac {1} {16}\ derecha) +\ cdots\\ &= 1 +\ frac {1} {2} +\ izquierda (\ frac {2} {4}\ derecha) +\ izquierda (\ frac {4} {8}\ derecha) +\ izquierda (\ frac {8} {16}\ derecha) +\ cdots\ end {align*}

    Entonces se puede ver que esto es\(1 + \frac{1}{2} +\frac{1}{2}+\frac{1}{2} +\frac{1}{2} +\cdots\) y así debe divergir 19.

    Hay muchas variaciones en la prueba de Oresme —por ejemplo, usar grupos de dos o tres. Una prueba bastante diferente se basa en la desigualdad

    \ begin {reunir*} e^x\ gt 1 + x\ qquad\ text {para $x\ gt 0$}\ end {reunir*}

    que se desprende inmediatamente de la serie Taylor para\(e^x\) dado en Teorema 3.6.7. A partir de esto podemos unir el exponencial de los números armónicos:

    \ begin {alinear*} e^ {h_n} &= e^ {1 +\ frac {1} {2} +\ frac {1} {3} +\ frac {1} {4} +\ cdots +\ frac {1} {n}}\\ &= e^1\ cdot e^ {1/2}\ cdot e^ {1/3}\ cdot e^ {1/4}\ cdots e^ {1/n}\\ &\ gt (1+1)\ cdot (1+1/2)\ cdot (1+1/3)\ cdot (1+1/4)\ cdots (1+1/n)\\ &=\ frac {2} {1}\ cdot\ frac {3} {2}\ cdot\ frac {4} {3}\ cdot\ frac {5} {4}\ cdots\ frac {n+1} {n}\\ &= n+1\ final {alinear*}

    Ya que\(e^{H_n}\) crece sin rebotes con\(n\text{,}\) la serie armónica diverge.

    El problema de Basilea

    El problema de determinar el valor exacto de la suma de la serie

    \ begin {reunir*}\ sum_ {n=1} ^\ infty\ frac {1} {n^2}\ end {reunir*}

    se llama el problema de Basilea. El problema lleva el nombre del pueblo natal de Leonhard Euler, quien lo resolvió. Se pueden usar series telescópicas para mostrar que esta serie debe converger. Observe que

    \ begin {align*}\ frac {1} {n^2} &\ lt\ frac {1} {n (n-1)} =\ frac {1} {n-1} -\ frac {1} {n}\ end {align*}

    De ahí que podamos encuadernar la suma parcial:

    \ begin {align*} s_k=\ sum_ {n=1} ^k\ frac {1} {n^2} &\ lt 1 +\ sum_ {n=2} ^k\ frac {1} {n (n-1)} &&\ text {evitar dividir por $0$}\\ &= 1 +\ sum_ {n=2} ^k\ left (\ frac {1} {-1} -\ frac {1} {n}\ derecha) &&\ text {que telescopian hacia}\\ &= 1 + 1 -\ frac {1} {k}\ end {alinear*}

    Así, a medida que\(k\) aumenta, la suma parcial\(S_k\) aumenta (la serie es una suma de términos positivos), pero siempre es menor que\(2\text{.}\) Así converge la secuencia de sumas parciales.

    Mengoli planteó el problema de evaluar la serie exactamente en 1644 y fue resuelta —no del todo rigurosamente— por Euler en 1734. Una prueba rigurosa tuvo que esperar otros 7 años. Euler utilizó algunas observaciones y manipulaciones extremadamente astutas de la función sinusoidal para demostrar que

    \ begin {alinear*}\ sum_ {n=1} ^\ infty\ frac {1} {n^2} &=\ frac {\ pi^2} {6}. \ end {alinear*}

    Usó la serie Maclaurin

    \ begin {alinear*}\ sin x &= 1 -\ frac {x^3} {6} +\ frac {x^5} {24} -\ cdots\ end {align*}

    y una fórmula de producto para seno

    \ begin {align}\ begin {split}\ sin x &= x\ cdot\ izquierda (1 -\ frac {x} {\ pi}\ derecha)\ cdot\ izquierda (1 +\ frac {x} {\ pi}\ derecha)\ cdot\ izquierda (1 -\ frac {x} {2\ pi}\ derecha)\ cdot\ izquierda (1 +\ frac {x} {2\ pi}\ derecha)\ cdot\ izquierda (1 -\ frac {x} {3\ pi}\ derecha)\ cdot\ izquierda (1 +\ frac {x} {3\ pi}\ derecha)\ cdots\\ &= x\ cdot\ izquierda (1 -\ frac {x^ 2} {\ pi}\ derecha)\ cdot\ izquierda (1 -\ frac {x^2} {4\ pi}\ derecha)\ cdot\ izquierda (1 -\ frac {x^2} {9\ pi}\ derecha)\ cdots\ end {split}\ label {eqn_sinprodformula}\ tag {\(\star\)}\ end {align}

    Extraer el coeficiente\(x^3\) de de ambas expansiones da el resultado deseado. La prueba de la fórmula del producto está mucho más allá del alcance de este curso. Pero fíjese que al menos los valores de los\(x\) cuales hacen que el lado izquierdo de (\(\star\)) cero, es decir,\(x=n\pi\) con\(n\) entero, son exactamente los mismos que los valores de los\(x\) cuales hacen que el lado derecho de (\(\star\)) cero 20.

    Este enfoque también se puede utilizar\(\sum_{n=1}^\infty n^{-2p}\) para calcular\(p=1,2,3,\cdots\) y demostrar que son múltiplos racionales 21 de\(\pi^{2p}\text{.}\) La serie correspondiente de poderes impares son significativamente más asquerosos y obtener expresiones de forma cerrada para ellos sigue siendo un famoso problema abierto.

    Opcional — Algunas pruebas

    En esta sección opcional proporcionamos pruebas de dos pruebas de convergencia. Utilizaremos repetidamente el hecho de que cualquier secuencia\(a_1\text{,}\)\(a_2\text{,}\)\(a_3\text{,}\)\(\cdots\text{,}\) de números reales que esté aumentando (es decir,\(a_{n+1}\ge a_n\) para todos\(n\)) y acotada (es decir, hay una constante\(M\) tal que\(a_n\le M\) para todos\(n\)) converge. No vamos a probar este hecho 22.

    Comenzamos con la prueba de comparación, y luego pasamos a la prueba de series alternas.

    Teorema 3.3.28 La Prueba de Comparación (declarado de nuevo)

    Dejar\(N_0\) ser un número natural y dejar\(K \gt 0\text{.}\)

    1. Si\(|a_n|\le K c_n\) por todos\(n\ge N_0\) y\(\sum\limits_{n=0}^\infty c_n\) converge, entonces\(\sum\limits_{n=0}^\infty a_n\) converge.
    2. Si\(a_n\ge K d_n\ge0 \) para todos\(n\ge N_0\) y\(\sum\limits_{n=0}^\infty d_n\) diverge, entonces\(\sum\limits_{n=0}^\infty a_n\) diverge.
    Prueba

    (a) Por hipótesis\(\sum_{n=0}^\infty c_n\) converge. Entonces basta con probar que\(\sum_{n=0}^\infty [Kc_n-a_n]\) converge, porque entonces, por nuestra Aritmética de series Teorema 3.2.9,

    \[ \sum_{n=0}^\infty a_n = \sum_{n=0}^\infty K c_n -\sum_{n=0}^\infty [Kc_n-a_n] \nonumber \]

    convergerán también. Pero para todos\(n\ge N_0\text{,}\)\(Kc_n-a_n\ge 0\) para que, por todas\(N\ge N_0\text{,}\) las sumas parciales

    \[ S_N = \sum_{n=0}^N [Kc_n-a_n] \nonumber \]

    aumentar con\(N\text{,}\) pero nunca se hace más grande que el número finito\(\sum\limits_{n=0}^{N_0} [Kc_n-a_n] + K \sum\limits_{n=N_0+1}^\infty c_n\text{.}\) Así que las sumas parciales\(S_N\) convergen como\(N\rightarrow\infty\text{.}\)

    b) Para toda\(N \gt N_0\text{,}\) la suma parcial

    \[ S_N = \sum_{n=0}^N a_n \ge \sum_{n=0}^{N_0} a_n + K\hskip-10pt\sum_{n=N_0+1}^N\hskip-10pt d_n \nonumber \]

    Por hipótesis,\(\sum_{n=N_0+1}^N d_n\text{,}\) y por lo tanto\(S_N\text{,}\) crece sin ataduras como\(N\rightarrow\infty\text{.}\) Así\(S_N\rightarrow\infty\) como\(N\rightarrow\infty\text{.}\)

    Teorema 3.3.29 Prueba de Serie Alternativa (declarado de nuevo)

    Dejar\(\big\{a_n\big\}_{n=1}^\infty\) ser una secuencia de números reales que obedece

    1. \(a_n\ge 0\)para todos\(n\ge 1\) y
    2. \(a_{n+1}\le a_n\)para todos\(n\ge 1\) (es decir, la secuencia es monótona decreciente) y
    3. \(\lim_{n\rightarrow\infty}a_n=0\text{.}\)

    Entonces

    \[ a_1-a_2+a_3-a_4+\cdots=\sum\limits_{n=1}^\infty (-1)^{n-1} a_n =S \nonumber \]

    converge y, por cada número natural\(N\text{,}\)\(S-S_N\) está entre\(0\) y (el primer término caído)\((-1)^N a_{N+1}\text{.}\) Aquí\(S_N\) está, como antes, la suma\(N^{\rm th}\) parcial\(\sum\limits_{n=1}^N (-1)^{n-1} a_n\text{.}\)

    Prueba

    \(2n\)Sea un número parejo natural. Entonces la suma\(2n^{\rm th}\) parcial obedece

    \ begin {alinear*} S_ {2n} &=\ overbrackets {(a_1-a_2)} ^ {\ ge 0} +\ overbrackets {(a_3-a_4)} ^ {\ ge 0} +\ cdots +\ overbrackets {(a_ {2n-1} -a_ {2n})} ^ {\ ge 0}\\ & le\\ overcorsé {(a_1-a__2)} ^ {\ ge 0} +\ overbrackets {(a_3-a_4)} ^ {\ ge 0} +\ cdots +\ overbrackets {(a_ {2n-1} -a_ {2n})} ^ {\ ge 0} +\ overbrackets {(a_ {2n+1} -a_ {2n+2})} ^ {\ ge 0}\\ & =S_ {2 (n+1)}\ end {align*}

    y

    \ begin {align*} S_ {2n} &=a_1- (\ overbrackets {a_2-a_3} ^ {\ ge 0}) - (\ overbrackets {a_4-a_5} ^ {\ ge 0}) -\ cdots -\ overbrackets {(a_ {2n-2} -a_ {2n-1})} ^ {\ ge 0} -\ overcorsé {a_ {2n-1})} ^ {\ ge 0} -\ overcorsé {a_ {2n-1})} ^ {\ ge n}} ^ {\ ge 0}\\ &\ le a_1\ end {alinear*}

    Entonces la secuencia\(S_2\text{,}\)\(S_4\text{,}\)\(S_6\text{,}\)\(\cdots\) de sumas parciales pares es una secuencia acotada, creciente y por lo tanto converge a algún número real\(S\text{.}\) ya que\(S_{2n+1} = S_{2n} +a_{2n+1}\)\(a_{2n+1}\) converge cero ya que\(n\rightarrow\infty\text{,}\) las sumas parciales impares\(S_{2n+1}\) también convergen a\(S\text{.}\) Eso \(S-S_N\)está entre\(0\) y (el primer término caído) ya\((-1)^N a_{N+1}\) se probó en §3.3.4.

    Ejercicios

    Etapa 1
    1

    Seleccione las siguientes series que diverjan por la prueba de divergencia.

    (A)\(\displaystyle\sum_{n=1}^\infty \frac{1}{n}\)

    (B)\(\displaystyle\sum_{n=1}^\infty \frac{n^2}{n+1}\)

    (C)\(\displaystyle\sum_{n=1}^\infty \sin n\)

    (D)\(\displaystyle\sum_{n=1}^\infty \sin (\pi n)\)

    2

    Seleccione la serie a continuación cuyos términos cumplan las condiciones para aplicar la prueba integral.

    (A)\(\displaystyle\sum_{n=1}^\infty \frac{1}{n}\)

    (B)\(\displaystyle\sum_{n=1}^\infty \frac{n^2}{n+1}\)

    (C)\(\displaystyle\sum_{n=1}^\infty \sin n\)

    (D)\(\displaystyle\sum_{n=1}^\infty \frac{\sin n+1}{n^2}\)

    3

    Supongamos que hay algún umbral después del cual una persona es considerada vieja, y ante la cual es joven.

    Que Olaf sea una persona mayor, y que Yuan sea joven.

    1. Supongamos que soy mayor que Olaf. ¿Soy viejo?
    2. Supongamos que soy más joven que Olaf. ¿Soy viejo?
    3. Supongamos que soy mayor que Yuan. ¿Soy joven?
    4. Supongamos que soy más joven que Yuan. ¿Soy joven?
    4

    A continuación se presentan gráficas de dos secuencias con términos positivos. Supongamos que las secuencias continúan como se muestra. Rellene la tabla con conclusiones que se puedan hacer a partir de la prueba de comparación directa, en su caso.

    image-518.svg

      si\(\sum a_n\) converge si\(\sum a_n\) diverge
    y si\(\{a_n\}\) es la serie roja entonces\(\sum b_n\)\(\Rule{2cm}{1pt}{0pt}\) entonces\(\sum b_n\)\(\Rule{2cm}{1pt}{0pt}\)
    y si\(\{a_n\}\) es la serie azul entonces\(\sum b_n\)\(\Rule{2cm}{1pt}{0pt}\) entonces\(\sum b_n\)\(\Rule{2cm}{1pt}{0pt}\)
    5

    Para cada par de series a continuación, decida si la segunda serie es una serie comparativa válida para determinar la convergencia de la primera serie, utilizando la prueba de comparación directa y/o la prueba de comparación de límites.

    1. \(\displaystyle\sum_{n=10}^{\infty} \frac{1}{n-1},\)en comparación con la serie divergente\(\displaystyle\sum_{n=10}^{\infty} \frac{1}{n}.\)
    2. \(\displaystyle\sum_{n=1}^{\infty} \frac{\sin n}{n^2+1},\)en comparación con la serie convergente\(\displaystyle\sum_{n=1}^{\infty} \frac{1}{n^2}.\)
    3. \(\displaystyle\sum_{n=5}^{\infty} \frac{n^3+5n+1}{n^6-2},\)en comparación con la serie convergente\(\displaystyle\sum_{n=5}^{\infty} \frac{1}{n^3}.\)
    4. \(\displaystyle\sum_{n=5}^{\infty} \frac{1}{\sqrt{n}},\)en comparación con la serie divergente\(\displaystyle\sum_{n=5}^{\infty} \frac{1}{\sqrt[4]n}.\)
    6

    Supongamos que\(a_n\) es una secuencia con\(\displaystyle\lim_{n \to \infty}a_n = \frac{1}{2}\text{.}\) ¿\(\displaystyle\sum_{n=7}^\infty a_n\)Converge o diverge, o no es posible determinar esto a partir de la información dada? ¿Por qué?

    7

    ¿Qué defecto invalida el siguiente razonamiento?

    P: Determinar si\(\displaystyle\sum_{n=1}^\infty \dfrac{\sin n}{n}\) converge o diverge.

    R: Primero, evaluaremos\(\displaystyle\lim_{n \to \infty} \dfrac{\sin n}{n}\text{.}\)

    • Nota\(\dfrac{-1}{n} \leq \dfrac{\sin n}{n} \leq \dfrac{1}{n}\) para\(n \ge 1\text{.}\)
    • Tenga en cuenta también que\(\displaystyle\lim_{n \to \infty}\frac{-1}{n}=\displaystyle\lim_{n \to \infty}\frac{1}{n}=0\text{.}\)
    • Por lo tanto, por el Teorema de Squeeze,\(\displaystyle\lim_{n \to \infty} \dfrac{\sin n}{n}=0\) también.

    Entonces, por la prueba de divergencia,\(\displaystyle\sum_{n=1}^\infty \dfrac{\sin n}{n}\) converge.

    8

    ¿Qué defecto invalida el siguiente razonamiento?

    P: Determinar si\(\displaystyle\sum_{n=1}^\infty \left(\sin(\pi n)+2\right)\) converge o diverge.

    R: Utilizamos la prueba integral. Let\(f(x)=\sin(\pi x)+2\text{.}\) Note siempre\(f(x)\) es positivo, ya que\(\sin(x)+2 \geq -1+2 =1\text{.}\) También,\(f(x)\) es continuo.

    \ begin {alinear*}\ int_1^\ infty [\ sin (\ pi x) +2]\, d {x} &=\ lim_ {b\ a\ infty}\ int_1^b [\ sin (\ pi x) +2]\, d {x}\\ &=\ lim_ {b\ a\ infty}\ left [\ left. -\ frac {1} {\ pi}\ cos (\ pi x) +2x\ derecha|_1^b\ derecha]\\ &=\ lim_ {b\ a\ infty}\ izquierda [-\ frac {1} {\ pi}\ cos (\ pi b) +2b +\ frac {1} {\ pi} (-1) -2\ derecha]\\ &=\ infty\ end {align*}

    Por la prueba integral, ya que la integral diverge, también\(\displaystyle\sum_{n=1}^\infty\left( \sin(\pi n)+2\right)\) diverge.

    9

    ¿Qué defecto invalida el siguiente razonamiento?

    P: Determinar si la serie\(\displaystyle\sum_{n=1}^\infty \dfrac{2^{n+1}n^2}{e^n+2n}\) converge o diverge.

    R: Queremos comparar esta serie con la serie\(\displaystyle\sum_{n=1}^\infty \dfrac{2^{n+1}}{e^n}\text{.}\) Tenga en cuenta que tanto esta serie como la serie en la pregunta tienen términos positivos.

    Primero, encontramos que\(\dfrac{2^{n+1}n^2}{e^n+2n} \gt \dfrac{2^{n+1}}{e^n}\) cuando\(n\) es suficientemente grande. La justificación de esta alegación es la siguiente:

    • Observamos que\(e^n(n^2-1) \gt n^2-1 \gt 2n\) por\(n\) suficientemente grande.
    • Por lo tanto,\(e^n \cdot n^2 \gt e^n+2n\)
    • Por lo tanto,\(2^{n+1}\cdot e^n \cdot n^2 \gt 2^{n+1}(e^n+2n)\)
    • Dado que\(e^n+2n\) y\(e^n\) son ambas expresiones que resultan ser positivas para los valores de\(n\) bajo consideración, podemos dividir ambos lados de la desigualdad por estos términos sin tener que voltear la desigualdad. Entonces,\(\dfrac{2^{n+1}n^2}{e^n+2n} \gt \dfrac{2^{n+1}}{e^n}\text{.}\)

    Ahora, afirmamos\(\displaystyle\sum_{n=1}^\infty \dfrac{2^{n+1}}{e^n}\) converge.

    Nota\(\displaystyle\sum_{n=1}^\infty \dfrac{2^{n+1}}{e^n}= 2\displaystyle\sum_{n=1}^\infty \dfrac{2^{n}}{e^n}= 2\displaystyle\sum_{n=1}^\infty \left(\dfrac{2}{e}\right)^n\text{.}\) Esta es una serie geométrica con\(r=\frac{2}{e}\text{.}\) Desde que\(2/e \lt 1\text{,}\) la serie converge.

    Ahora, por la Prueba de Comparación Directa, concluimos que\(\displaystyle\sum_{n=1}^\infty \dfrac{2^{n+1}n^2}{e^n+2n}\) converge.

    10

    ¿Cuáles de las siguientes series están alternando?

    (A)\(\displaystyle\sum_{n=1}^\infty \sin n\)

    (B)\(\displaystyle\sum_{n=1}^\infty \frac{\cos(\pi n)}{n^3}\)

    (C)\(\displaystyle\sum_{n=1}^\infty \frac{7}{(-n)^{2n}}\)

    (D)\(\displaystyle\sum_{n=1}^\infty \frac{(-2)^n}{3^{n+1}}\)

    11

    Dé un ejemplo de una serie convergente para la cual la prueba de relación no es concluyente.

    12

    Imagina que estás tomando un examen, y momentáneamente olvidas exactamente cómo funciona la desigualdad en la prueba de ratio. Recuerdas que hay una proporción, pero no recuerdas qué término va arriba; recuerdas que hay algo acerca de que el límite sea mayor o menor que uno, pero no recuerdas qué camino implica convergencia.

    Explicar por qué

    \[ \lim_{n \to \infty}\left|\frac{a_{n+1}}{a_{n}}\right| \gt 1 \nonumber \]

    o, equivalentemente,

    \[ \lim_{n \to \infty}\left|\frac{a_n}{a_{n+1}}\right| \lt 1 \nonumber \]

    debe significar que la suma\(\sum\limits_{n=1}^\infty a_n\) diverge (en lugar de converger).

    13

    Dar un ejemplo de una serie\(\displaystyle\sum_{n=a}^\infty a_n\text{,}\) con una función\(f(x)\) tal que\(f(n)=a_n\) para todos los números enteros\(n\text{,}\) tal que:

    • \(\displaystyle\int_a^\infty f(x)\,\, d{x}\)diverge, mientras
    • \(\displaystyle\sum_{n=a}^\infty a_n\)converge.
    14 (✳)

    Supongamos que desea utilizar la Prueba de Comparación de Límite en la serie\(\displaystyle \sum_{n=0}^{\infty} a_n\) donde\(\displaystyle a_n = \frac{2^n+n}{3^n+1}\text{.}\)

    Anote una secuencia\(\{b_n\}\) tal que\(\displaystyle \lim\limits_{n\to\infty} \frac{a_n}{b_n}\) exista y sea distinta de cero. (No es necesario realizar la Prueba de Comparación de Límites)

    15 (✳)

    Decidir si cada una de las siguientes afirmaciones es verdadera o falsa. Si es falso, proporcione un contraejemplo. Si es cierto proporcionar una breve justificación.

    1. Si\(\displaystyle\lim_{n\rightarrow\infty}a_n=0\text{,}\) entonces\(\sum\limits_{n=1}^{\infty} a_n\) converge.
    2. Si\(\displaystyle\lim_{n\rightarrow\infty}a_n=0\text{,}\) entonces\(\sum\limits_{n=1}^{\infty} (-1)^{n\mathstrut} a_n\) converge.
    3. Si\(0\le a_n \le b_n\) y\(\sum\limits_{n=1}^{\infty} b_n\) diverge, entonces\(\sum\limits_{n=1}^{\infty} a_n\) diverge.
    Etapa 2
    16 (✳)

    ¿\(\displaystyle \sum_{n=2}^\infty \frac{n^2}{3n^2+\sqrt n}\)Converge la serie?

    17 (✳)

    Determinar, con explicación, si la serie\(\displaystyle \sum_{n=1}^\infty \frac{5^k}{4^k+3^k}\) converge o diverge.

    18 (✳)

    Determinar si la serie\(\displaystyle\sum_{n=0}^\infty\frac{1}{n+\frac{1}{2}}\) es convergente o divergente. Si es convergente, encuentra su valor.

    19 (✳)

    ¿Las siguientes series convergen o divergen? \(\displaystyle\sum_{k=1}^\infty\frac{1}{\sqrt{k}\sqrt{k+1}}\)

    20

    Evalúe las siguientes series, o demuestre que diverge:\(\displaystyle\sum_{k=30}^\infty 3(1.001)^k\text{.}\)

    21

    Evalúe las siguientes series, o demuestre que diverge:\(\displaystyle\sum_{n=3}^\infty \left(\frac{-1}{5}\right)^n\text{.}\)

    22

    ¿Las siguientes series convergen o divergen? \(\displaystyle\sum_{n=7}^\infty \sin(\pi n)\)

    23

    ¿Las siguientes series convergen o divergen? \(\displaystyle\sum_{n=7}^\infty\cos(\pi n)\)

    24

    ¿Las siguientes series convergen o divergen? \(\displaystyle\sum_{k=1}^\infty \frac{e^k}{k!}\text{.}\)

    25

    Evalúe las siguientes series, o demuestre que diverge:\(\displaystyle\sum_{k=0}^\infty\frac{2^k}{3^{k+2}}\text{.}\)

    26

    ¿Las siguientes series convergen o divergen? \(\displaystyle\sum_{n=1}^\infty\frac{n!n!}{(2n)!}\text{.}\)

    27

    ¿Las siguientes series convergen o divergen? \(\displaystyle\sum_{n=1}^\infty\frac{n^2+1}{2n^4+n}\text{.}\)

    28 (✳)

    Demuestre que la serie\(\displaystyle\sum_{n=3}^\infty \frac{5}{n(\log n)^{3/2}}\) converge.

    29 (✳)

    Encuentra los valores\(p\) para los que\(\displaystyle{\sum_{n=2}^\infty \frac{1}{n(\log n)^p}}\) converge la serie.

    30 (✳)

    ¿\({\displaystyle\sum_{n=1}^\infty\frac{e^{-\sqrt{n}}}{\sqrt{n}}}\)Converge o diverge?

    31 (✳)

    Utilice la prueba de comparación (no la prueba de comparación límite) para mostrar si la serie

    \[ \sum_{n=2}^{\infty} \frac{\sqrt{3 n^2 - 7}}{n^{3}} \nonumber \]

    converge o diverge.

    32 (✳)

    Determinar si la serie\(\displaystyle\sum_{k=1}^\infty\frac{ \root{3}\of{k^4+1} } {\sqrt{k^5+9}}\) converge.

    33 (✳)

    ¿\(\displaystyle\sum_{n=1}^\infty\frac{n^4 2^{n/3}}{(2n+7)^4}\)Converge o diverge?

    34 (✳)

    Determinar, con explicación, si cada una de las siguientes series convergen o divergen.

    1. \(\displaystyle\sum_{n=1}^\infty\frac{1}{\sqrt{n^2+1}}\)
    2. \(\displaystyle\sum_{n=1}^\infty\frac{n\cos(n\pi)}{2^n}\)
    35 (✳)

    Determinar si la serie

    \[ \sum_{k=1}^\infty\frac{k^4-2k^3+2}{k^5+k^2+k} \nonumber \]

    converge o diverge.

    36 (✳)

    Determinar si cada una de las siguientes series convergen o divergen.

    1. \(\displaystyle\sum_{n=2}^\infty\frac{n^2+n+1}{n^5-n}\)
    2. \(\displaystyle\sum_{m=1}^\infty\frac{3m+\sin\sqrt{m}}{m^2}\)
    37

    Evalúe las siguientes series, o demuestre que diverge:\(\displaystyle\sum_{n=5}^\infty \frac{1}{e^n}\text{.}\)

    38 (✳)

    Determinar si la serie\(\displaystyle\sum_{n=2}^\infty\frac{6}{7^n}\) es convergente o divergente. Si es convergente, encuentra su valor.

    39 (✳)

    Determinar, con explicación, si cada una de las siguientes series convergen o divergen.

    1. \(1+\frac{1}{3}+\frac{1}{5}+\frac{1}{7}+\frac{1}{9}+\cdots\text{.}\)
    2. \({\displaystyle\sum_{n=1}^\infty \frac{2n+1}{2^{2n+1}}}\)
    40 (✳)

    Determinar, con explicación, si cada una de las siguientes series converge o diverge.

    1. \({\displaystyle \sum_{k=2}^\infty \frac{\root{3}\of{k}}{k^2-k}}\text{.}\)
    2. \({\displaystyle \sum_{k=1}^\infty \frac{k^{10}10^k(k!)^2}{(2k)!}}\text{.}\)
    3. \({\displaystyle \sum_{k=3}^\infty \frac{1}{k(\log k) (\log\log k)}}\text{.}\)
    41 (✳)

    Determinar si la serie\(\displaystyle\sum_{n=1}^\infty\frac{n^3-4}{2n^5-6n}\) es convergente o divergente.

    42 (✳)

    ¿Cuál es el valor más pequeño de\(N\) tal que la suma parcial\(\displaystyle\sum_{n=1}^N\frac{(-1)^n}{n\cdot 10^n}\) se aproxima\(\displaystyle\sum_{n=1}^\infty\frac{(-1)^n}{n\cdot 10^n}\) dentro de una precisión de\(10^{-6}\text{?}\)

    43 (✳)

    Se sabe que\(\displaystyle \sum_{n=1}^\infty \frac{(-1)^{n-1}}{n^2} = \frac{\pi^2}{12}\) (no hay que mostrar esto). Encuentra\(N\) para que\(S_N\text{,}\) la suma\(N^{\rm th}\) parcial de la serie, satisfaga\(| \frac{\pi^2}{12} - S_N | \le 10^{-6}\text{.}\) Asegúrate de decir por qué tu método se puede aplicar a esta serie en particular.

    44 (✳)

    La serie\(\displaystyle \sum_{n=1}^\infty \frac{(-1)^{n+1}}{(2n+1)^2}\) converge a algún número\(S\) (no tienes que probarlo). De acuerdo con el Teorema de Estimación de Serie Alternante, cuál es el valor más pequeño\(N\) para el cual la suma\(N^{\rm th}\) parcial de la serie está a lo sumo\(\frac1{100}\) alejada de\(S\text{?}\) Para este valor de\(N\text{,}\) escribir la suma\(N^{\rm th}\) parcial de la serie.

    Etapa 3

    Una serie de fenómenos siguen aproximadamente una distribución llamada ley de Zipf. Se discuten algunas de ellas en las preguntas 52 y 53.

    45 (✳)

    Determinar, con explicación, si las siguientes series convergen o divergen.

    1. \(\displaystyle\sum_{n=1}^\infty\frac{n^n}{9^n n!}\)
    2. \(\displaystyle\sum_{n=1}^\infty\frac{1}{n^{\log n}}\)
    46 (✳)

    a) Demostrar que\(\displaystyle \int_2^\infty\frac{x+\sin x}{1+x^2}\ \, d{x}\) diverge.

    (b) Explique por qué no se puede concluir que\(\displaystyle\sum\limits_{n=1}^\infty \frac{n+\sin n}{1+n^2}\) diverge de la parte (a) y la Prueba Integral.

    c) Determinar, con explicación, si\(\displaystyle\sum\limits_{n=1}^\infty \frac{n+\sin n}{1+n^2}\) converge o diverge.

    47 (✳)

    Mostrar que\(\displaystyle\sum\limits_{n=1}^\infty\frac{e^{-\sqrt{n}}}{\sqrt{n}}\) converge y encuentra un intervalo de longitud\(0.05\) o menor que contenga su valor exacto.

    48 (✳)

    Supongamos que la serie\(\displaystyle\sum\limits_{n=1}^\infty a_n\) converge y que\(1 \gt a_n\ge 0\) para todos\(n\text{.}\) Probar que la serie\(\displaystyle\sum\limits_{n=1}^\infty \frac{a_n}{1-a_n}\) también converge.

    49 (✳)

    Supongamos que la serie\(\sum\limits_{n=0}^{\infty}(1-a_n)\) converge, donde\(a_n \gt 0\) para\(n=0,1,2,3,\cdots\text{.}\) Determine si la serie\(\sum\limits_{n=0}^\infty 2^n a_n\) converge o diverge.

    50 (✳)

    Supongamos que la serie\(\displaystyle\sum_{n=1}^\infty\frac{na_n-2n+1}{n+1}\) converge, donde\(a_n \gt 0\) para\(n = 1, 2, \cdots\text{.}\) Es la siguiente serie

    \ begin {reunir*} -\ log a_1 +\ sum_ {n=1} ^\ infty\ log\ Grande (\ frac {a_n} {a_ {n+1}}\ Grande)\ end {reunir*}

    convergente? Si tu respuesta es NO, justifica tu respuesta. Si tu respuesta es SÍ, evalúa la suma de la serie\(-\log a_1 + \sum\limits_{n=1}^\infty \log\big(\frac{a_n}{a_{n+1}}\big)\text{.}\)

    51 (✳)

    Demostrar que si\(a_n\ge 0\) para todos\(n\) y si la serie\(\displaystyle\sum_{n=1}^\infty a_n\) converge, entonces la serie\(\displaystyle\sum_{n=1}^\infty a^2_n\) también converge.

    52

    Supongamos que la frecuencia de uso de palabras en un idioma tiene el siguiente patrón:

    La\(n\) -ésima palabra más utilizada representa el\(\dfrac{\alpha}{n}\) porcentaje del total de palabras utilizadas.

    Entonces, en un texto de 100 palabras, esperamos que la palabra más utilizada aparezca\(\alpha\) veces, mientras que la segunda palabra más utilizada debería aparecer alrededor de\(\frac{\alpha}{2}\) veces, y así sucesivamente.

    Si los libros escritos en este idioma usan palabras\(20,000\) distintas, entonces la palabra más utilizada representa aproximadamente qué porcentaje del total de palabras utilizadas?

    53

    Supongamos que los tamaños de las ciudades en un país se adhieren al siguiente patrón: si la ciudad más grande tiene población\(\alpha\text{,}\) entonces la\(n\) -ésima ciudad más grande tiene población\(\frac{\alpha}{n}\text{.}\)

    Si la ciudad más grande de este país tiene 2 millones de personas y la ciudad más pequeña tiene 1 persona, entonces la población de todo el país es\(\sum_{n=1}^{2 \times 10^6}\frac{2\times 10^6}{n}\text{.}\) (Para muchos\(n\) en esta suma no\(\frac{2\times 10^6}{n}\) es un entero. Ignorar eso.) Evalúe esta suma aproximadamente, con un error de no más de 1 millón de personas.

    1. Los autores deben ser un poco más cuidadosos al hacer una declaración tan general. Si bien es cierto que no es prudente aproximar una serie divergente tomando\(N\) términos con\(N\) grandes, hay casos en los que se puede obtener una muy buena aproximación tomando\(N\) términos con\(N\) pequeños! Por ejemplo, el teorema del resto de Taylor nos muestra que cuando la\(n^{\rm th}\) derivada de una función\(f(x)\) crece muy rápidamente con\(n\), los polinomios de Taylor de grado\(N\), con\(N\) grandes, pueden dar malas aproximaciones de\(f(x)\), mientras que los polinomios Taylor de grado uno o dos todavía puede proporcionar muy buenas aproximaciones de\(f(x)\) cuando\(x\) es muy pequeño. Como ejemplo de ello, uno de los triunfos de la electrodinámica cuántica, a saber, el cálculo del momento magnético anómalo del electrón, depende precisamente de esto. Se predijeron varias cantidades importantes utilizando los primeros términos de series de potencias divergentes. Cuando esas cantidades se midieron experimentalmente, las predicciones resultaron ser increíblemente precisas.
    2. El campo del análisis asintótico a menudo hace uso de los primeros términos de series divergentes para generar soluciones aproximadas a los problemas; esto, junto con los cálculos numéricos, es una de las técnicas más importantes en matemáticas aplicadas. En efecto, hay todo un libro maravilloso (que, desafortunadamente, es demasiado avanzado para la mayoría de los estudiantes de Cálculo 2) dedicado a jugar con series divergentes llamadas, como era de esperar, “Divergent Series” de G.H. Hardy. Esto no debe confundirse con la serie “Divergente” de V. Roth ambientada en una Chicago distópica postapocalíptica. Esa última serie diverge bastante dramáticamente de los temas matemáticos, mientras que la primera no tiene una adaptación cinematográfica (todavía).
    3. Hemos discutido el contrapositivo varias veces en las notas del CLP, pero no hace daño volver a discutirlo aquí (o que el lector busque rápidamente la nota al pie de página relevante en la Sección 1.3 del texto CLP-1). En todo caso, dada una declaración de la forma “Si A es verdadera, entonces B es verdadera” el contrapositivo es “Si B no es cierto, entonces A no es cierto”. Las dos declaraciones entre comillas son lógicamente equivalentes —si una es verdadera, entonces también lo es la otra. En el presente contexto tenemos “Si (\(\sum a_n\) converges) then (\(a_n\) converges to \(0\)).” The contrapositive of this statement is then “If (\(a_n\) does not converge to 0) then (\(\sum a_n\) does not converge).”
    4. Esta serie converge a la constante de Apéry\(1.2020569031\dots\text{.}\) The constant is named for Roger Apéry (1916–1994) who proved that this number must be irrational. This number appears in many contexts including the following cute fact — the reciprocal of Apéry's constant gives the probability that three positive integers, chosen at random, do not share a common prime factor.
    5. Latín para “Una vez que se hacen los cambios necesarios”. Esta frase todavía se usa un poco, pero en estos días los matemáticos tienden a escribir algo equivalente en inglés. En efecto, el inglés es más o menos la lengua franca para la publicación matemática. Quidquid erit.
    6. Esta serie, vista como una función de\(p\), se llama la función zeta de Riemann,\(\zeta(p)\text{,}\) o la función zeta de Euler-Riemann. Es sumamente importante por sus conexiones con números primos (entre muchas otras cosas). Efectivamente Euler demostró que\ (\ zeta (p) =\ sum_ {n=1} ^\ infty\ frac {1} {n^p}
      =\ prod_ {\ text {P prime}}\ left (1 - {\ rm P} ^ {-p}\ right) ^ {-1}\ text {.}\). Riemann mostró las conexiones entre los ceros de esta función (sobre números complejos\(p\)) y la distribución de números primos. Podría decirse que el problema sin resolver más famoso en matemáticas, la hipótesis de Riemann, se refiere a las ubicaciones de ceros de esta función.
    7. Podríamos ir aún más lejos y ver qué pasa si incluimos poderes de\(\log(\log(n))\) y otras funciones más exóticas de crecimiento lento.
    8. Regresa y escanea rápidamente el Teorema 3.3.5; para aplicarlo necesitamos demostrar que\(\frac{1} {n^2+2n+3}\) is positive and decreasing (it is), and then we need to integrate \(\int \frac{1}{x^2+2x+3}\, d{x}\text{.}\) To do that we reread the notes on partial fractions, then rewrite \(x^2+2x+3 = (x+1)^2+2\) and so \(\int_1^\infty \frac{1}{x^2+2x+3}\, d{x} = \int_1^\infty \frac{1}{(x+1)^2+2}\, d{x} \cdots\) and then arctangent appears, etc etc. Urgh. Okay — let's go back to the text now and see how to avoid this.
    9. Para entender esto considera cualquier serie\(\sum_{n=1}^\infty a_n\text{.}\) We can always cut such a series into two parts — pick some huge number like \(10^6\text{.}\) Then \(\sum_{n=1}^\infty a_n = \sum_{n=1}^{10^6} a_n + \sum_{n=10^6+1}^\infty a_n \text{.}\) The first sum, though it could be humongous, is finite. So the left hand side, \(\sum_{n=1}^\infty a_n\text{,}\) is a well-defined finite number if and only if \(\sum_{n=10^6+1}^\infty a_n\text{,}\) is a well-defined finite number. The convergence or divergence of the series is determined by the second sum, which only contains \(a_n\) for “large” \(n\text{.}\)
    10. El símbolo “\(\gg\)” means “much larger than”. Similarly, the symbol “\(\ll\)” means “much less than”. Good shorthand symbols can be quite expressive.
    11. Esto es muy similar a cómo calculamos los límites en el infinito camino de regreso cerca del comienzo de CLP-1.
    12. Realmente, muy tedioso. Y pensabas que algunos de esos cálculos de fracciones parciales eran malos...
    13. El lector interesado tal vez desee echar un vistazo a “La aproximación de Stirling”, que dice que\(n!\approx \sqrt{2\pi n}\left(\frac {n}{e}\right)^{n}\text{.}\)
    14. Veremos más adelante, en el Teorema 3.5.13, que la función\(\sum_{n=0}^\infty a n x^{n-1}\) is indeed the derivative of the function \(\sum_{n=0}^\infty a x^n\text{.}\) Of course, such a statement only makes sense where these series converge — how can you differentiate a divergent series? (This is not an allusion to a popular series of dystopian novels.) Actually, there is quite a bit of interesting and useful mathematics involving divergent series, but it is well beyond the scope of this course.
    15. También veremos más adelante, en el Teorema 3.5.13, que la función\(\sum_{n=0}^\infty \frac{a}{n+1} x^{n + 1}\) is indeed an antiderivative of the function \(\sum_{n=0}^\infty a x^n\text{.}\)
    16. Podría ser una buena idea revisar el inicio del §2.3 en este punto.
    17. Al menos si nuestra mesa es lo suficientemente fuerte.
    18. El lector interesado deberá utilizar su buscador favorito para leer más sobre el vínculo entre esta serie y los armónicos musicales. También puedes encontrar interesantes vínculos entre la serie Harmonic y el llamado “problema del jeep” y también el problema de apilar una torre de dominó para crear un voladizo que no se vuelque.
    19. El argumento de agrupación puede generalizarse más y el lector interesado debe buscar la prueba de condensación de Cauchy.
    20. Sabiendo que los lados izquierdo y derecho de (\(\star\)) are zero for the same values of \(x\) is far from the end of the story. Two functions \(f(x)\) and \(g(x)\) having the same zeros, need not be equal. It is certainly possible that \(f(x)=g(x)*A(x)\) where \(A(x)\) is a function that is nowhere zero. The interested reader should look up the Weierstrass factorisation theorem.
    21. Buscador tu camino a la “función zeta de Riemann”.
    22. Es una manera de declarar una propiedad del sistema de números reales llamada “integridad”. El lector interesado debe utilizar su motor de búsqueda favorito para buscar “integridad de los números reales”.

    This page titled 3.3: Pruebas de Convergencia is shared under a CC BY-NC-SA 4.0 license and was authored, remixed, and/or curated by Joel Feldman, Andrew Rechnitzer and Elyse Yeager via source content that was edited to the style and standards of the LibreTexts platform; a detailed edit history is available upon request.